Mastering Proficiency Skills in English

0 downloads 0 Views 1MB Size Report
Question tags are used after affirmative or negative sentences to check information or ... Further, a hyphen (-) is obligatory between the number and the noun. ... As for the second blank, when explaining the position of something, we say that it lies .... and would/wouldn't + BARE INFINITIVE (b) in the main clause are used ...
Jovana Dimitrijević-Savić Branka Milenković Jelena Danilović

MAPS Mastering Proficiency Skills in English PRVO IZDANJE

april 2010. Kragujevac

MAPS Mastering Proficiency Skills in English Jovana Dimitrijević-Savić, Branka Milenković, Jelena Danilović Izdavač Filološko-umetnički fakultet Kragujevac, Jovana Cvijića bb Za izdavača Slobodan Štetić Recenzenti prof. dr Savka Blagojević prof. dr Biljana Mišić-Ilić Lektori Jasmina Teodorović Tatjana Grujić Jelena Krstanović Likovno-grafička oprema Goran Kojadinović Tehnički urednik Slavoljub Radojević Štampa Tiraž

CONTENTS CONTENTS ..............................................................................................................3 PREDGOVOR ...........................................................................................................4 A GUIDE THROUGH THE BOOK ..............................................................................6 GLOSSARY ...............................................................................................................9 GRAMMAR / VOCABULARY..................................................................................11 TEST 1 ...............................................................................................................12 TEST 2 ...............................................................................................................32 TEST 3 ...............................................................................................................52 TEST 4 ...............................................................................................................72 TEST 5 ...............................................................................................................92 READING COMPREHENSION ..............................................................................113 TEST 1 .............................................................................................................114 TEST 2 .............................................................................................................117 TEST 3 .............................................................................................................120 TEST 4 .............................................................................................................123 TEST 5 .............................................................................................................125 WRITING .............................................................................................................129 SUGGESTED ESSAY TOPICS ............................................................................135 FURTHER PRACTICE ............................................................................................136 REFERENCES .......................................................................................................137

Mastering Proficiency Skills in English

PREDGOVOR Pored mnogobrojnih knjiga, priručnika i kompilacija testova koji su se pojavili na tržistu poslednjih godina iznova se javljaju nove potrebe i odatle proizilaze novi metode u iznalaženju najboljeg vida testiranja za posebne potrebe učenika engleskog kao stranog jezika. Uslovljeni su nivoima i kompetencijama poznavanja engleskog jezika pa shodno tome imaju za cilj da pripreme učenike i testiraju njihov nivo znanja kako bi ih osposobili za uspešno pohađanje nastave i usvajanje jezika na akademskom nivou, kao i za sticanje međunarodnih diploma. Priručnik je osmišljen kao prateći materijal koji se, uz gramatike i odgovarajuće rečnike, može koristiti u pripremi za polaganje testova za proveru znanja i jezičkih veština na nivou B2 engleskog jezika (prema Zajedničkom evropskom referentnom okviru – Common European Framework of Reference), kao i za pripremu budućih studenata anglistike za prijemne ispite na fakultetima u Srbiji. Autori priručnika, nastavnici na Odseku za anglistiku, vodeći se analizom rezultata sa prethodnih prijemnih ispita sastavili su testove koji objedinjuju sve komponente prisutne u formi prijemnog ispita. Ponuđene su tri vrste testova: razumevanje teksta (reading comprehension), potom test gramatike i vokabulara (multiple-choice test) i test pismenog izražavanja (composition). Reading comprehension testira razumevanje tekstova raznovrsne tematike, dužine 450-500 reči. Multiple choice test objedinjuje 30 pitanja koja pokrivaju poznavanje gramatike i leksike, kao i osnovnih idioma i fraza. Composition segment nudi tri teme a od kandidata se očekuje koherentno i smisaono izlaganje na datu temu dužine jedne strane. Svaki odeljak u ovoj zbirci osmišljen je sa ciljem da pomogne kandidatu u izradi testova detaljnim pratećim objašnjenjima kako tačnih, tako i netačnih odgovora. Gramatičke sekvence su objašnjene tako što se nadovezuju na samu obradu materije iz gramatika, oslanjajući se prvenstveno na kontekst koji uslovljava određene konstrukcije i izbor leksike. Takođe su ponuđena i objašnjenja za tačne/netačne odgovore u reading comprehension testu koja ne samo što otkrivaju rešenja, već upućuju kandidate na sam metod rešavanja ovakvih testova i karakteristične formulacije pitanja. Composition test se ne oslanja samo na gramatički ispravne konstrukcije rečenica, već je akcenat na sadržini i koherentnom izlaganju na datu temu gde ozbiljnost gramatičkih grešaka ne predstavlja prepreku razumevanju teksta. Autori su se opredelili za američki varijetet engleskog jezika u zbirci iz razloga što su mladi sve naklonjeniji ovoj varijanti, pa se ukazuje potreba da se i u priručnicima ovog tipa nađe kontekst koji zastupa ovu varijantu. page | 4

Predgovor

U priručniku su ponuđena četiri sastava sa prijemnog ispita održanog jula 2008. godine na Filološko-umetničkom fakultetu u Kragujevcu, od kojih su dva zadovoljila kriterijum, a dva ne. Kako bi kandidati mogli što potpunije i kvalitetnije da se pripreme za pisanje sastava svaki sastav kandidata na ispitu propraćen je kratkim obrazloženjem koje ističe zadovoljene/nezadovoljene elemente veštine pisanja i ukazuje na odeljke koji bi trebalo da se preformulišu kako bi tekst bio razumljiv. Na samom kraju priručnika se nalaze ponuđene teme za pisanje sastava. Bitno je napomenuti da se na početku zbirke nalazi glosar gramatičkih pojmova koji ima za cilj da kandidate upozna sa terminologijom koja će biti zastupljena u objašnjenjima, a pre svakog segmenta ponuđeni su kratak uvod i sugestije kako pristupiti rešavanju svakog testa. Sveobuhvatnoj pripremi doprinose i posebne gramatičke reference (Points of interest) koje budućim kandidatima omogućuju dodatan osvrt na gramatičke i leksičke kategorije. Zahvalnost dugujemo recenzentima, prof. dr Savki Blagojević i prof. dr Biljani Mišić-Ilić, koje su nam omogućile da ova zbirka bude realizovana, kao i Jasmini Teodorović, Tatjani Grujić i Jeleni Krstanović na konstruktivnim savetima i sugestijama čime su doprinele kvalitetu zbirke. Autori snose celokupnu odgovornost u pogledu nedostataka i eventualnih propusta koji su načinjeni prilikom sastavljanja zbirke. Autori

page | 5

Mastering Proficiency Skills in English

A GUIDE THROUGH THE BOOK To the students… The manner in which a book is used is as important as the content of the book. As a result it is essential that some guidelines are provided for students using this book as additional, preparatory material. While doing the tests it would be advisable that you cover the explanatory notes. Only after completing all 30 questions in the multiple choice section should you check whether your answers were correct and analyze the grammar sections that need revision. In the multiple choice tests you should consider the content of the entire sentence and not focus on the gaps individually in the order in which they appear. Sometimes the second or the third gap will suggest the correct answer for the previous one(s). It is very important that you understand the content and the construction of the whole sentence as this will help you decide which tense, word form or other grammatical item is necessary in order to complete the sentence correctly. In the reading comprehension section you should consider the wider context, paying attention to sentences that both precede and follow the item in question. Analyze the whole paragraph a specific question refers to and carefully consider each and every provided answer, at the same time relating it to the original text. Special care should be taken with the questions that test specific words which refer to a word, phrase, clause or idea in the text, as the answer may be found before or after them. When writing a one-page composition, choose the topic that suits your sphere of interests best and that provides you with many ideas for writing. It is important that these ideas are organized so that you introduce the topic, develop it and finally conclude the composition, summarizing the main ideas. While the actual length of a composition may vary depending on student handwriting, the topic must be fully treated by all. The sentences should be clear, understandable and connected to one another. Because the compositions are graded by a passing or failing mark it is clear that you should pay attention to producing grammatically correct sentences as much as possible. Where grammar mistakes are numerous and create a problem in reading and understanding of the text, the composition will receive a failing mark.

page | 6

A Guide Through the Book

To the teachers… It is our duty, as teachers, to help our students on their journey through education. They need to master essential learning skills and strategies and it is we who must guide them in this endeavor. Therefore, it will be useful for teachers who will help future students preparing for entrance exams at Departments of English language and literature in Serbia, as well as other B2 level tests (CEFR) to become familiar with the authors’ aim in writing this book. At the very beginning it is necessary to introduce the students to the glossary of grammatical terms which will be used in the explanatory notes throughout the book. It is also necessary to teach them to utilize the concepts they will encounter in grammar books. The explanatory notes that follow the multiple choice tests provide a key to the questions. As the explanations in the notes consistently refer to the context of the entire sentence, it is of the utmost importance that students are made fully aware that the most advantageous approach is the one in which each sentence is considered as a whole and the context in which each blank occurs is given careful consideration. These notes stress the importance of the context of the entire sentence. Whenever decomposing a sentence the teacher should invoke in the student a constant need to closely inspect what precedes and follows each blank. Only in this manner will they learn to relate the parts of the sentence to one another and be able to grasp the meaning of the whole construction. A content-based approach, recognized by second language educators, fosters the belief that this approach enables the students to connect language and content instruction meaningfully (J.M. O’Malley & L.V. Pierce, 1996). The explanatory notes make evident which grammatical concept/item needs to be revised if the answer is not correct and the References at the end of the book suggest the literature that could be consulted. Points of interest, appearing at the end of explanatory notes, are additional guidelines for those who need further explanation of certain grammatical items. In the reading comprehension tests, which are not an uncommon means of language assessment, the focus is on the speed with which the students are able to read and grasp the main ideas of the text. It is vital that they are able to find the information and answer the questions correctly by decomposing the paragraphs from which the questions have been extracted. Once again, paying attention to the understanding of the content, they need to concentrate on the entire segment of the text. The questions test the understanding of constructions, reference words, synonyms/synonymous expressions and negation. Therefore the teachers should instruct students to employ different ways of approaching these questions. page | 7

Mastering Proficiency Skills in English

Writing implies a process comprised of different skills. The nature of the required text must be considered alongside the objectives of writing. These objectives are accompanied by the scoring criteria that clearly identify the (un)satisfactory level of composition. Instead of analytical scoring we have opted to apply a holistic scoring method, as we agree with O’Malley and Pierce’s (1996:142) assertion that “the total quality of written text is more than the sum of its components”. We view writing as a representation of an integrated whole and therefore we grade students’ compositions as passing/failing compositions. Also taken into account is the students’ prior knowledge and the possible discrepancy among them in terms of the level of language proficiency with which they approach the writing task. The essay topic proposed is just as significant as the criteria selected for essay assessment. The choice of topics was carefully treated and takes account of the students’ level of language proficiency, awareness and knowledge of the skill of writing, as well as their interests. The topics additionally preset grammatical frames with reference to specific time, sentence and verbal constructions, consequently placing specific demands upon the students. Thus the variety of topics selected covers descriptive, narrative and contemporary disputable issues. Time reference consists of past, present and future structures, which also entail hypothetical situations, and requires of the students to express personal impressions, beliefs, fears, hopes and wishes. We hope that these tests will provide useful material for self-study, as well as for the entrance exams or other B2 level tests preparation. Additionally, they provide different assessment methods that can be employed by teachers, striving to meet students’ needs.

page | 8

Glossary

GLOSSARY abstract noun ...................................................... apstraktna imenica adjective ............................................................. pridev adverb ................................................................. prilog auxiliary verb ...................................................... pomoćni glagol bare / to-infinitive .............................................. krnji infinitiv/ infinitiv sa to cardinal / ordinal numbers ................................. glavni / redni brojevi collective noun .................................................... zbirna imenica collocation .......................................................... kolokacija (uobičajeni spoj reči) comparative ........................................................ komparativ compound............................................................ složenica conditional sentences ........................................ uslovne rečenice conjunction .......................................................... veznik consonant ............................................................ suglasnik countable / uncountable noun ........................... brojiva / nebrojiva imenica definite / indefinite article .................................. određeni / neodređeni član demonstrative adjective / pronoun .................. pokazni pridev / zamenica emphasis / emphasize ......................................... isticanje / isticati Englishlike nonwords / nonexpressions ............ nepostojeće eng. reči koje imaju prizvuk postojećih gerund .................................................................. gerund imperative ........................................................... zapovedni način indirect (reported) speech .................................. neupravni govor infinitive .............................................................. infinitiv intransitive / transitive verb .............................. neprelazni / prelazni glagol irregular / regular verb........................................ nepravilan / pravilan glagol inversion ............................................................. inverzija main clause / subordinate clause ....................... glavna / zavisna klauza modal verb........................................................... modalni glagol noun .................................................................... imenica noun / verb phrase .............................................. imenička / glagolska fraza object .................................................................. objekat passive voice ....................................................... pasiv past participle ..................................................... prošli particip perfect participle ................................................ particip perfekta present participle ................................................ sadašnji particip page | 9

Mastering Proficiency Skills in English

perfect infinitive .................................................. infinitiv perfekta personal pronoun ................................................ lična zamenica plural .................................................................... množina possessive adjective / pronoun .......................... prisvojni pridev / zamenica phrasal verb ......................................................... frazalni (frazni) glagol prefix .................................................................... prefiks preposition ........................................................... predlog pronoun .............................................................. zamenica quantifier ............................................................. kvantifikator question tags ....................................................... upitni privesci reflexive pronoun ................................................ povratna zamenica relative pronoun .................................................. odnosna zamenica sequence of tenses .............................................. slaganje vremena singular ................................................................. jednina subjunctive .......................................................... konjuktiv syllable ................................................................. slog suffix ..................................................................... sufiks superlative .......................................................... superlativ verb ..................................................................... glagol vowel ................................................................... samoglasnik

page | 10

GRAMMAR / VOCABULARY This section requires students to complete sentences by choosing either A, B, C, or D. Students are advised to read each sentence carefully, paying attention to all the blanks (if there is more than one), and to the grammatical and/or lexical items which are being tested. In this way, as students progress through the tests, they should become increasingly aware of the importance of a logical, step-by-step approach to doing this kind of exercises and the percentage of correct answers selected should also increase.

Mastering Proficiency Skills in English

TEST 1 1. April Fool’s Day comes on __________. a) first April b) the April first c) April of the first d) the first of April

2. __________ they are sad – neither of the brothers has __________ own bedroom. a) No way / their b) No chance / theirs c) No wonder / his d) No surprise / its

3. __________, bottled water __________ tap water to a slight degree. a) All in all / differentiates b) On the whole / differs from c) Generally / contrasts from d) Most / contradicts

page | 12

Grammar/Vocabulary

1. CORRECT ANSWER:

d) the first of April

Dates are written by using ordinal numbers which are always preceded by the definite article, e.g. the first, the second, the thirteenth…followed by the preposition of and the month in question. Therefore the only correct answer is (d). POINTS OF INTEREST: ordinal numbers, definite article

2. CORRECT ANSWER:

c) No wonder / his

The choice of the phrase in the first blank is governed by the need for it to express lack of surprise in order to be consistent with the meaning of the second part of the sentence. a) No way can be ruled out immediately since it is an emphatic way of saying no. b) No chance and d) No surprise are Englishlike nonexpressions. Only c) No wonder is correct in this context. The second part of the sentence starts with neither followed by a singular verb has and thus requires an appropriate possessive adjective (note the object …own bedroom), that is c) his. Therefore, all the other options, a) their, b) theirs or d) its, have to be ruled out. POINTS OF INTEREST: negative word neither, possessive adjectives

3. CORRECT ANSWER:

b) On the whole / differs from

The explanation for the second blank is provided first as it governs the meaning of the sentence. The second blank requires a verb meaning ‘be unlike in some way’, that is b) differ. a) to differentiate is used to indicate a difference between things; c) to contrast cannot be followed by the preposition from. Finally, d) to contradict is used specifically when there is a disagreement between statements, pieces of evidence, stories, etc. which is not the case in this context. While any one of the options offered for the first blank, apart from d) most which can never be used on its own at the beginning of a sentence (compare mostly), is possible, only (b) gives the appropriate solution to both blanks. POINTS OF INTEREST: verb + preposition

page | 13

Mastering Proficiency Skills in English

4. __________ people travel, it is not a rarity that they experience __________ shock and need some time to adjust. a) Whatever / cultivating b) Whatsoever / cultural c) Wherever / culture d) Whenever / cultivated

5. Would you like __________ more salad perhaps? ~ Yes, please and could you pass me __________ vinegar? a) any / b) some / the c) a little / d) little / the

6. My __________ sister and I discussed the __________ of our late aunt Lily’s life. a) eldest / odds and ends b) elder / ups and downs c) oldest / ins and outs d) older / pros and cons

page | 14

Grammar/Vocabulary

4. CORRECT ANSWER:

c) Wherever / culture

Only c) Wherever and d) Whenever are possible solutions to the first blank due to their meaning. a) Whatever is used for referring to anything or everything that is available, needed or wanted; b) Whatsoever is used after a noun group to emphasize a negative statement, e.g. I have no doubt whatsoever. The second blank requires the noun c) culture which collocates with the noun shock denoting ‘the nervous or confused feeling that people sometimes get when they arrive in a place that has a very different culture from their own’. The remaining options do not make sense in this context. POINTS OF INTEREST: conjunctions, collocations

5. CORRECT ANSWER:

b) some / the

Any of the three options offered for the first blank, excluding d) little, are possible solutions. a) any is commonly used in questions and negatives; b) some can be used in offers, when we expect people to answer ‘yes’, or want to encourage them to do so; e.g. Could I have some brown rice, please? c) a little is almost synonymous with some; d) little is inappropriate because, as opposed to a little, it has a rather negative meaning, suggesting ‘not as much as one would like’. Further, although uncountable nouns are commonly used with zero article, the second blank requires the use of the definite article since the implication is ‘you and I both know which I mean’ (the hearer can see the vinegar in question). POINTS OF INTEREST: some/any, little/a little, definite article

6. CORRECT ANSWER:

b) elder / ups and downs

Any one of the four options offered for the first blank is possible since b) elder and a) eldest can be used instead of d) older and c) oldest to talk about the order of birth of siblings. They are only used attributively, that is, before nouns. My elder/older sister has just got engaged. She’s five years elder older than me. The second blank requires an expression meaning ‘a mixture of good things and bad things’, or b) ups and downs. The other options have to be ruled out because of their meaning: a) odds and ends refers to a group of small things that are all different and are not valuable or important; c) the ins and outs of a situation are all the detailed points and facts to do with it, while d) the pros and cons of something are its advantages and disadvantages.

page | 15

Mastering Proficiency Skills in English

7. He can barely stand on his feet, __________? It’s two months since he __________ a car crash but his legs are still __________. a) can’t he / had / tender b) can’t he / has had / fragile c) can he / had / weak d) can he / has had / unsteady

8. The report provides __________ evidence of the political __________ in Georgia and the writer’s provocative comment __________ its possible consequences. a) frightening / crises / on b) frightened / crisis / of c) frightening / crisis / about d) frightened / crises / to

9. Daniel’s parents realized that he was addicted __________ Internet gambling and __________ professional __________. a) from / looked / advise b) to / sought / advice c) to / searched / advice d) from / turned to / advise

page | 16

Grammar/Vocabulary

7. CORRECT ANSWER:

c) can he / had / weak

Question tags are used after affirmative or negative sentences to check information or ask for agreement. If the verb in the main clause has an auxiliary verb, the question tag has the same auxiliary. After clauses containing negative words such as barely, never, no, nobody, hardly, scarcely or little, non-negative tags are applied, e.g. It’s no good, isn’t is it? This immediately eliminates a) and b) can’t he as potential correct answers. The second blank comes after since, a conjunction of time referring to, in this case, the specific time in the past when the accident happened, so only the Simple Past Tense form of the verb a) and c) had is possible here. Finally, all adjectives offered for the third blank have a similar meaning and are equally appropriate but only (c) meaningfully completes all blanks. POINTS OF INTEREST: question tags, Present Perfect/Past Simple

8. CORRECT ANSWER:

c) frightening / crisis / about

The first blank requires an adjective with the meaning ‘having this effect’, i.e. a PRESENT PARTICIPLE functioning as an adjective, not an adjective with the meaning ‘affected in this way’, i.e. a PAST PARTICIPLE (compare interesting/interested, fascinating/fascinated, boring/bored). Therefore, b) and d) frightened can be ruled out. The second blank requires the use of a noun in the singular, since its possible consequences are mentioned, that is b) or c) crisis. a) and d) crises is the plural form of crisis. There are many other words similar to crisis-crises which have special plural forms such as basis-bases, diagnosis-diagnoses, analysis-analyses, oasis-oases, hypothesis-hypotheses, thesis-theses, emphasis-emphases, metamorphosis-metamorphoses. The third blank requires a preposition which is commonly used after the noun comment. Either a) on or c) about is an adequate solution, yet only answer (c) fits all blanks. POINTS OF INTEREST: adjectives ending in –ing/–ed, plural of nouns

9. CORRECT ANSWER:

b) to / sought / advice

The adjective addicted is commonly followed by the preposition to. In the first blank it is, therefore, possible to disqualify a) and d) from. What is missing in the second blank is a suitable verb in the Simple Past Tense meaning ‘try to find’. b) sought is the Simple Past Tense form of the verb to seek, which is an irregular verb. Both a) looked and c) searched would have to be used with the preposition for to convey the meaning in question; further, d) turned to is inappropriate in this context because you turn to somebody for help, counsel, etc. Finally, the third blank requires a noun, not the verb advise, so both b) and c) advice are a possibility, yet only answer (b) appropriately completes all three blanks. POINTS OF INTEREST: adjective + preposition, confusable words page | 17

Mastering Proficiency Skills in English

10. We had __________ meal at the Majestic and enjoyed __________ tremendously. a) four course / them b) a four-courses / in it c) a four-course / it d) four courses / in them

11. Beatrice had __________ that she had to call her cousin, a __________, and ask for help. a) such terrible a headache / psychiatrist b) so terrible headache / psychic c) such terrible headache / physicist d) so terrible a headache / physician

12. The Olympic Stadium in Beijing __________ a bird’s nest and lies north __________ the city center. a) reminds of / from b) takes after / at c) looks like / of d) reminds on / to

page | 18

Grammar/Vocabulary

10. CORRECT ANSWER:

c) a four-course / it

The first blank in the sentence requires the ‘noun as adjective’ structure with the meaning ‘a meal that includes four courses’. When numbers are used in these structures, the second part of the compound is normally in the singular, e.g. a ten-pound note a twenty-liter can a two-month tour Further, a hyphen (-) is obligatory between the number and the noun. Therefore, the only correct option is (c). Moreover, the second blank comes after the transitive verb enjoy which does not require a preposition before its direct object. It is therefore possible to immediately eliminate answers b) in it and d) in them. Since the verb enjoy refers to the aforementioned meal, the singular pronoun c) it meaningfully completes the sentence, not a) them. POINTS OF INTEREST: compound adjectives, transitive verbs

11. CORRECT ANSWER:

d) so terrible a headache / physician

The first blank requires either of the following structures: such + a + ADJECTIVE + NOUN or so + ADJECTIVE + a + NOUN; their meaning is the same. It is, therefore, immediately possible to rule out a) such terrible a headache due to the incorrect word order and c) such terrible headache which treats headache as an uncountable noun and omits the indefinite article. The same is true for b) so terrible headache. The second blank should be completed with a noun whose meaning is synonymous with ‘a medical doctor’. Both b) psychic and c) physicist are inappropriate here owing to their meaning. The former means ‘having mysterious powers which let one read the minds of others, or see into the future’ and the latter ‘a person who deals with physics professionally.’ a) a psychiatrist is ‘a doctor who treats people with mental illnesses’ which could be regarded as a potential good choice. Finally, d) a physician is another term for a medical doctor, which makes (d) an appropriate solution to both blanks. POINTS OF INTEREST: so/such (a), indefinite article

12. CORRECT ANSWER:

c) looks like / of

The first blank requires a VERB + PREPOSITION structure describing the impression the Olympic Stadium gives, that is, what it c) looks like. a) reminds of, as well as d) reminds on, which contains an unsuitable preposition, is not a potential solution because the verb remind is a transitive verb, i.e. has to be followed by an object. b) takes after is also inappropriate here because the Olympic Stadium is not a human being, and thus, cannot be said to resemble somebody (compare In looks Susan takes after her dad). As for the second blank, when explaining the position of something, we say that it lies north/south/east/west of some place. page | 19

Mastering Proficiency Skills in English

13. In the morning the alarm clock will __________, waking the __________ teens, who had spent the night out. a) turn on / sleepy b) go off / drowsy c) go on / asleep d) turn off / lazy

14. __________ at a __________ of land and sea, Singapore is an Asian melting pot of cultures. a) Situating / crossroad b) Having situated / crossroads c) Having been situated / crossroad d) Situated / crossroads

15. What __________ ugly jeans! They __________ designed by Victoria Beckham. a) an / will have been b) - / would have been c) an / must have been d) - / can’t have been

page | 20

Grammar/Vocabulary

13. CORRECT ANSWER:

b) go off / drowsy

The first blank needs to be completed with a suitable phrasal verb which collocates with the noun alarm clock and means ‘to make a sudden loud noise’, that is b) go off. Item c) go on is a possible, though unlikely solution, whereas a) turn on and d) turn off are inappropriate as they imply a person’s adjusting the controls of a piece of equipment. Further, any of the three adjectives a) sleepy, b) drowsy or d) lazy are potential good choices for the second blank. The remaining item c) asleep has to be ruled out since it is only used after a verb, not in front of a noun, e.g. John hoped that his fiancée would be asleep when he got home. (*asleep fiancée) POINTS OF INTEREST: phrasal verbs

14. CORRECT ANSWER:

d) Situated / crossroads

The first blank requires a participle form which replaces a SUBJECT + PASSIVE VERB structure, Singapore (which) is situated…, that is a PAST PARTICIPLE, found in d) Situated. a) Situating, the PRESENT PARTICIPLE, is not possible here since this verb is a transitive one and there is no object used after it. b) Having situated, the PERFECT PARTICIPLE ACTIVE, is grammatically unacceptable for the same reason – lack of object. c) Having been situated, the PERFECT PARTICIPLE PASSIVE, is not a possible solution either because it is used to emphasize that an action has been completed before the beginning of another one, which is not the case in this context. The second blank requires a noun meaning ‘the place where two roads meet and cross each other’, which is called a crossroads, the noun which has the identical form for singular and plural. This immediately disqualifies crossroad in (a) and (c). POINTS OF INTEREST: participles, nouns having the same form for sing./pl.

15. CORRECT ANSWER:

d) --- / can’t have been

The noun jeans is a plural noun and can, therefore, never be preceded by the indefinite article. The same is true for a number of nouns such as trousers, scissors, scales, headquarters, pajamas, leggings, glasses, binoculars, clothes, panties, paratroops, stairs, goods, outskirts, earnings, belongings. This immediately eliminates a) and c) an as potential correct answers. Furthermore, what is missing in the second blank is an appropriate past tense structure since a statement is being made about the designer of the aforesaid jeans. Therefore, b) would have been, which is commonly found in main clauses in conditional sentences type 3, has to be ruled out, while the other items are all possible here: a) will have been denotes an assumption about a past action, c) must have been denotes certainty about a past action, and d) can’t have been denotes the impossibility of an action to have taken place in the past. Only (d), however, meaningfully completes both the first and the second blank. POINTS OF INTEREST: plural nouns, modals in the past page | 21

Mastering Proficiency Skills in English

16. The school bus __________ and let the children out __________ their eager parents. a) pulled up / to meet b) pulled in / meeting c) pulled up / meet d) pulled in / to have met

17. Cuba still struggles with crime and poverty __________ its reputation as a __________ paradise for tourists. a) although / caring b) in spite of / carefree c) even though / careful d) despite / careless

18. Feeling that his days __________, he composed a will. a)

were numbered

b) would be counted c) will have been numbered d) would have been counted

page | 22

Grammar/Vocabulary

16. CORRECT ANSWER:

a) pulled up / to meet

The first blank requires an appropriate phrasal verb whose meaning is ‘stop’. Both options offered are potential solutions, with a slight difference in meaning: a) and c) pulled up means ‘to bring to a stop (often abrupt)’, while b) and d) pulled in ‘to arrive at a destination or to come to a stop’. In order to find the correct answer for the second blank, it is necessary to keep in mind that the verb let is commonly followed by an OBJECT + BARE INFINITIVE, e.g. let me see or an OBJECT + PREPOSITIONAL PHRASE + to-INFINITIVE as in a) to meet (let the children out to meet). All three options apart from (a) are, therefore, ungrammatical for the second blank. POINTS OF INTEREST: phrasal verbs, the verb let

17. CORRECT ANSWER:

b) in spite of / carefree

The first blank is followed by a noun phrase its reputation. Therefore, the potential correct choices are either b) in spite of or d) despite, as they are commonly followed by noun phrases. a) although and c) even though have to be ruled out because they commonly precede a clause (subject and finite verb) which is not the case in this sentence. The second blank requires an adjective to modify the noun paradise. Of the four options b) carefree best fits the context because it denotes ‘the lack of worries, troubles, or responsibilities’. The other options do not meaningfully fit the context. POINTS OF INTEREST: although/in spite of, word formation by means of suffixes

18. CORRECT ANSWER:

a) were numbered

The blank requires the expression ‘somebody’s days are numbered’ which means that ‘someone is not likely to continue in a particular situation or to exist much longer’. This immediately disqualifies b) would be counted and d) would have been counted. Since the sentence begins with the present participle feeling, which substitutes Since/As/Because he felt…, the expression in question has to be in the Past Simple Tense due to the rule of the Sequence of Tenses. What is more, this is the reason why item c) will have been numbered, the Future Perfect form, is not a possible solution. POINTS OF INTEREST: phrases (idioms), Sequence of Tenses

page | 23

Mastering Proficiency Skills in English

19. Having __________ the table, the children __________ down to eat. a) set / set b) sat / set c) set / sat d) sat / sat

20. A large percentage of the archeological sites __________ fascinating, __________ the museums are rather uninteresting. a) is / while b) are / however c) are / whereas d) is / wherein

21. The tourists insisted __________ the bus so they could __________ the famous Blarney Stone. a) the driver would stop / spot b) that the driver should stop / look at c) on the driver to stop / view d) the driver’s stopping / observe

page | 24

Grammar/Vocabulary

19. CORRECT ANSWER:

c) set / sat

The correct expression missing in the first blank is to set the table, that is, to prepare the table for a meal by putting plates and cutlery on it. This immediately disqualifies b) and d) sat, the Past Simple form of the verb to sit. Moreover, the appropriate verb in the Past Simple Tense for the second blank is c) and d) sat, not a) and b) set which is the Past Simple form of the verb set.

20. CORRECT ANSWER:

c) are / whereas

A percentage of modifies the subject of this sentence the archaeological sites and the verb we should use therefore needs to be in the plural, not in the singular. This disqualifies a) and d) is for the first blank. Based on the meaning of the sentence and the balanced positive and negative characteristics that the two different parts of the sentence require, the second blank should be completed with a conjunction that introduces the contrast. a) while, besides indicating that an action is taking place at the same time of another, is also used to show the difference between two situations, places, etc. which is the meaning we need in this case; b) however is used when a fact or some information that seems surprising or is in disagreement with what has been said is added, but it is not a conjunction and is therefore inappropriate in this context; c) whereas is used to say that although something is true of one thing, it is not true of another, and is synonymous with while; d) wherein is a relative pronoun used in literary texts meaning ‘in which place/thing’, e.g. I’m looking for a riding school wherein we could learn the art of horsemanship. Only item (c) meaningfully completes both blanks. POINTS OF INTEREST: subject – verb agreement, conjunctions

21. CORRECT ANSWER:

b) that the driver should stop / look at

What is missing in the first blank is an appropriate complement of the verb insist which can either be followed by a that-CLAUSE or on + GERUND. It is therefore possible to rule out immediately both a) the driver would stop and d) the driver’s stopping. c) on the driver to stop includes on + to-INFINITIVE and is also a wrong option. As for the second blank, any of the three answers excluding c) view, would be possible, as the meaning of this verb is ‘to walk around the house, garden, or exhibition in order to look at it thoroughly or examine it‘. POINTS OF INTEREST: gerund

page | 25

Mastering Proficiency Skills in English

22. Had you turned left at the traffic light, we __________ in this dreary place right now. It’s all your __________. a) wouldn’t have been stuck / mistake b) wouldn’t be stuck / fault c) weren’t stuck / mistake d) would have been stuck / fault

23. Tara is still __________ her new working schedule, spending a lot of time __________. a) used to / outdoor b) getting used to / outdoors c) use to / out of doors d) being used to / out of door

24. It’s time Jack’s parents __________ more responsibility on his shoulders. He’s __________ of age next month. a) lay / growing b) laid / coming c) lie / getting d) lied / turning

page | 26

Grammar/Vocabulary

22. CORRECT ANSWER:

b) wouldn’t be stuck / fault

This sentence contains a combination of types 2 and 3 conditional sentences. Had you turned left… means the same as If you had turned left… The Past Perfect Tense in the if-CLAUSE and would/wouldn’t + BARE INFINITIVE (b) in the main clause are used when there is some (un)fulfilled past action which has an effect on the present (note the use of …right now). The second blank requires a noun which is commonly used when you blame somebody for something. The correct answer is not mistake, as in (a) and (c), but fault. POINTS OF INTEREST: conditional sentences

23. CORRECT ANSWER:

b) getting used to / outdoors

The first blank requires the usage of used as an adjective meaning ‘accustomed’, that is to be/become/get used to phrase. Tara is struggling with the schedule which is new to her, so to describe this ongoing process we can say that she is getting/growing/becoming used to it (b). As the choice of either c) use to or d) being used to would not result in the correct expression, these can be disqualified immediately. a) used to signals that Tara has had the schedule for some time, so it is no longer strange or new to her and is, therefore, inappropriate. Further, the second blank requires an adverb meaning ’outside’, that is, b) outdoors. While a) outdoor is an adjective, c) out of doors and d) out of door are not expressions used in English. POINTS OF INTEREST: be/get used to, adjectives/adverbs

24. CORRECT ANSWER:

b) laid / coming

It is time is a phrase which can either be followed by the to-INFINITIVE or PAST SUBJUNCTIVE. This rules out c) lie, the bare infinitive form, as a potential correct choice. The remaining three options a) lay, b) laid and d) lied can easily be confused, so it is important to stress the difference between them. b) laid is a transitive irregular verb (lay-laid-laid), meaning ‘to put something down’. Both a) lay and d) lied are the Past Simple Tense forms of an intransitive verb to lie which can function as an irregular (lie-lay-lain) or a regular verb (lie-lied-lied). As an irregular verb, to lie means ‘to be in horizontal position or get into one’, while as a regular verb it means ‘to say something untrue’. Neither (a) nor (d) can be used to complete the first blank meaningfully. Further, of the four options offered for the second blank, only (b) is appropriate here because it is a part of the phrase ‘to come of age’, that is ‘legally become an adult’. POINTS OF INTEREST: irregular verbs lie/lay, subjunctive page | 27

Mastering Proficiency Skills in English

25. Jenny simply doesn’t fit in with the crowd with all her exceptional qualities. She’s one __________. a) bird in the nest b) jailbird c) early bird d) rare bird

26. The actors were impatient for the rehearsal to begin. “__________! We’ll start __________ everyone arrives”, said the director. a) Hold yourselves / as long as b) Hang on to your hats / as soon as c) Hang on / as long as d) Hold your horses / as soon as

27. Forensic experts searched Mike’s house and found __________ evidence. Now it’s up to the jury to reach __________ unanimous verdict of his guilt or innocence. a) discriminating / a b) incriminating / an c) discriminating / an d) incriminating / a

page | 28

Grammar/Vocabulary

25. CORRECT ANSWER:

d) rare bird

The expression which best fits the blank is (d) in the sense of ‘somebody very unusual’. b) jailbird is used to refer to ‘somebody who has spent a lot of time in prison’. c) early bird and a) bird in the nest are also inappropriate due to their respective meanings. The former is used to describe ‘someone who likes to get up early in the morning’, whereas the latter is not an idiom and only has a literal meaning.

26. CORRECT ANSWER:

d) hold your horses / as soon as

Only c) hang on and d) hold your horses could be used to complete the first blank, as both mean ‘wait or be patient’. a) hold yourselves and b) hang on to your hats are expressions that are not a possibility since they are not idiomatic expressions in English and only have a literal meaning. The second blank requires the use of a conjunction denoting that ‘something will happen immediately after something else has happened’, that is b) and d) as soon as, rather than a) or c) as long as, which are commonly used to express conditions. POINTS OF INTEREST: idioms, conjunctions

27. CORRECT ANSWER:

d) incriminating / a

The first blank requires an adjective close in meaning to ‘proving that someone is guilty of something’, that is b) and d) incriminating. a) and c) discriminating is used for ‘describing someone who has the ability to recognize things which are of good quality’ and is therefore inappropriate in this context. What is missing in the second blank is an article. The choice between a) and d) a and b) and c) an depends on pronunciation, not spelling; an is used before a vowel sound, even if the vowel is not represented in spelling by the letters or any combination of these. Consequently, d) a is the correct answer, not b) an because unanimous is pronounced /ju:ˈnænɪməs/. Note: an H-bomb /ən ˈeɪʧbɒm/ a UFO /ə ˌju:efˈəʊ/

a university /ə ˌju:nɪˈvɜ:sɪtɪ/

an MA /ən ˌem'eɪ/

page | 29

Mastering Proficiency Skills in English

28. In 1912 the Titanic left the shores of Southampton and headed towards New York on its maiden __________. It collided with an iceberg and many people __________ in the cold waters of the Atlantic. a) travel / suffocated b) voyage / drowned c) journey / stiffened d) trip / choked

29. How __________ produce such __________ fine jewelry? a) did ancient Egyptians succeed in / b) were ancient Egyptians able to / a c) did ancient Egyptians manage to / d) could ancient Egyptians / a

30. Some fifty __________ of people consider a bright __________ star, or meteor, to be an unforgettable __________. a) per cents / falling / site b) per cent / failing / site c) per cents / shining / sight d) per cent / shooting / sight

page | 30

Grammar/Vocabulary

28. CORRECT ANSWER:

b) voyage / drowned

The correct expression, which completes the first blank, is maiden voyage, meaning the first official journey of a ship. a) travel is a general noun used for the activity of travelling; c) journey is an act of travelling, especially to a place that is far away, e.g. bus/train/car journey; d) trip denotes travelling somewhere and coming back, especially when you stay in a place for a short time, e.g. business/shopping/school trip. As for the second blank, a) suffocated, b) drowned and d) choked are all similar, denoting ‘inability to breathe’, while c) stiffened can be ruled out right away because of its meaning ‘to stop moving, with muscles suddenly becoming tense, e.g. because you feel afraid or angry’. b) drowned, however, implies dying under water, and thus meaningfully completes both blanks.

29. CORRECT ANSWER:

c) did ancient Egyptians manage to / ---

The first blank requires the use of an appropriate verbal phrase expressing the amazing ability of ancient Egyptians. It could be completed by either b) were able to, c) did manage to or d) could. The remaining option a) succeeded in is one of many prepositional verbs followed by GERUND, but not to-INFINITIVE as is the case in this sentence. In order to find the correct solution to the second blank, it is necessary to keep in mind that the noun jewelry is uncountable, so it can never be preceded by the indefinite article. POINTS OF INTEREST: verb + preposition + gerund, uncountable nouns

30. CORRECT ANSWER:

d) per cent / shooting / sight

Firstly, per cent is both the singular and plural form of this noun, which immediately disqualifies a) and c) per cents as potential correct answers. Secondly, only b) failing is inappropriate for the second blank due to its meaning, while all other options are possible solutions. Finally, the third blank requires a noun denoting something we see, that is c) and d) sight, rather than a) and b) site, ‘a plot of ground used for a particular purpose or the place where an important event happened’. Note that these two nouns, sight and site, are pronounced the same (linguistically speaking, they are homophones). POINTS OF INTEREST: nouns with the same form for sing./pl., homophones page | 31

Mastering Proficiency Skills in English

TEST 2 1. That was __________ big breakfast that they __________ us at the hotel! a) - / served b) one / supplied c) a / ordered d) the / delivered

2. __________ apartment is right opposite the town’s cemetery. Their children are not __________ of ghosts, however. a) Matthew’s and Jane’s / frightened b) Matthew’s and Jane / scared c) Matthew and Jane’s / afraid d) Matthew and Janes’ / terrified

3. That evening, __________, was certainly the __________ of my trip. a) meanwhile / light b) nonetheless / height c) nevertheless / lightning d) however / highlight

page | 32

Grammar/Vocabulary

1. CORRECT ANSWER:

c) a / ordered

Normally we do not use any articles before meals. Likewise, the indefinite article (a/an) is not used before names of meals, except when these are preceded by an adjective. Since this is the case in the sentence, it is therefore possible to immediately rule out (a) and (d). Item b) one, however, is also appropriate here since it can be used instead of a/an to emphasize the following adjective or expression. The second blank requires a suitable transitive verb. While a) served, c) ordered and d) delivered would all be correct in this context, b) supplied is inappropriate because it is commonly followed by an OBJECT + with. Still, only (c) meaningfully completes both blanks. POINTS OF INTEREST: articles

2. CORRECT ANSWER:

c) Matthew and Jane’s / afraid

The first blank requires an appropriate possessive form. Since the second sentence introduces their children, it becomes obvious that Matthew and Jane are a couple and that the apartment in question belongs to them. When two or more people own something together, make only the second (or final) person’s name possessive. Therefore, the only correct option is (c). As for the second blank, each of the four items is a possible solution since their meaning is ‘feel fear’. POINTS OF INTEREST: possessive case

3. CORRECT ANSWER:

d) however / highlight

The item needed to complete the first blank should emphasize the fact that the second point contrasts with the first. While b) nonetheless, c) nevertheless and d) however are appropriate here, a) meanwhile must be ruled out because it is used to introduce an aspect of a particular situation that is completely opposite to the one previously mentioned. For the second blank, only d) highlight is possible as its meaning is ‘the most interesting/exciting part’. The other options do not make sense in this context.

page | 33

Mastering Proficiency Skills in English

4. At a wedding __________, a bride and groom promise to love, honor and cherish __________ forever. a) procession / them b) service / themselves c) ceremony / one another d) celebration / the other

5. __________ the teachers nor the students remembered what they were supposed to do __________ an earthquake drill. a) Neither / during b) Either / at c) Both / while d) All / with

6. I have nothing against environmentally friendly goods. ~ __________. a) So do I. b) I do too. c) Neither me. d) Neither do I.

page | 34

Grammar/Vocabulary

4. CORRECT ANSWER:

c) ceremony / one another

All options offered for the first blank apart from a) procession are potential correct answers since all of them collocate with the noun wedding. Item (a) denotes ‘a group of people who are walking, riding, or driving in line as part of a public event.’ The second blank should be filled in with a reciprocal pronoun, that is c) one another, used for talking about actions that involve two or more people together in an identical way. Therefore, the remaining options must be excluded as inappropriate. POINTS OF INTEREST: reciprocal pronouns

5. CORRECT ANSWER:

a) Neither / during

Neither…nor is a fixed structure used to join two negative ideas. Therefore, all the other options can immediately be eliminated. Further, the second blank requires a suitable preposition to indicate the meaning ‘while the drill was taking place’. c) while has to be ruled out right away since it is a conjunction, that is, it joins clauses into sentences. Both b) at and d) with are also inappropriate because they cannot express the aforementioned intended meaning; finally, a) during is the correct option. POINTS OF INTEREST: neither…nor, conjunctions

6. CORRECT ANSWER:

d) Neither do I

The item missing in the blank is a response to the first, negative sentence, meaning ‘also not’. Answers (a) and (b) are, therefore, inappropriate. Instead, neither or nor could be used at the beginning of a sentence, followed by an inverted word order: AUXILIARY VERB + SUBJECT, so only option (d) is appropriate here. POINTS OF INTEREST: parallel responses, inversion

page | 35

Mastering Proficiency Skills in English

7. Drug trafficking __________ by no __________ than 60 per cent this year. a) have increased / less b) did increase / fewer c) increased / fewer d) has increased / less

8. She returned __________ home, __________ the early hours __________ Saturday morning. a) - / in / of b) to / at / in c) - / at / on d) to / in / at

9. Children __________ 16 are not __________ inside the casino. a) beneath / to allow b) below / to be allowed c) underneath / to let d) under / to be let

page | 36

Grammar/Vocabulary

7. CORRECT ANSWER:

d) has increased / less

The Present Perfect Tense is commonly used with an incomplete period of time (note… this year), implying that the action happened at some undefined time during this period. This narrows the options down to either a) have increased or d) has increased. Since the noun drug trafficking is uncountable, the best choice is (d). As for the second blank, no less than is used before an amount to emphasize that it is larger than you expected. POINTS OF INTEREST: Present Perfect/Past Simple, little/few

8. CORRECT ANSWER:

a) --- / in / of

The noun home can be preceded by a verb of motion and no preposition, e.g. return home. This immediately disqualifies both b) and d) to as potential correct answers. The second and third blank complete the expression in the early hours of. POINTS OF INTEREST: prepositions of place and time

9. CORRECT ANSWER:

d) under / to be let

The first blank requires an appropriate preposition to indicate that children haven’t reached the age of 16 yet, that is, they are d) under 16. All other options are not appropriate in this context. As for the second blank, the choice between (a) and (c) to-INFINITIVE and (b) and (d) PASSIVE INFINITIVE forms is governed by the fact that the blank is followed by a prepositional phrase and not an object which is expected after the verbs to let, that is, to allow. It is, therefore, obvious that the children are not the ones who perform the action in this sentence and that a passive structure is missing. Only the answer (d) meaningfully completes both blanks. POINTS OF INTEREST: prepositions, passive infinitive, transitive verbs

page | 37

Mastering Proficiency Skills in English

10. The detective looked at him in __________ and continued pacing __________. a) trust / here and there b) belief / up and down c) disbelief / back and forth d) distrust / off and on

11. Never __________ to see __________, coyotes, bison and other wildlife in the National Park back then. a) was I able / moose and reindeer b) am I able / mooses and reindeers c) had I been able / mooses and reindeers d) am I being able / moose and reindeer

12. Only after the __________ rehearsal, did the whole __________ of ‘Macbeth’ decide to go to a pub and __________. a) final / crew / lose out b) clothes / party / get together c) dress / cast / loosen up d) general / troupe / get lost

page | 38

Grammar/Vocabulary

10. CORRECT ANSWER:

c) disbelief / back and forth

The first blank requires a noun suitable for completion of the verbal part of the sentence looked at him in… - either c) disbelief or d) distrust. Items a) trust and b) belief cannot be used in this way. Further, the second blank should be completed with a phrase meaning ‘moving repeatedly in one direction and then in the opposite one’, that is, c) back and forth or b) up and down. The remaining options do not make sense in this context due to their meaning: a) here and there implies that an action is happening ‘in several different places’, while d) off and on means ‘occasionally, now and again, for a part of a period of time’. Only (c), however, is a well-suited option for both blanks. POINTS OF INTEREST: word formation by means of prefixes, prepositional phrases

11. CORRECT ANSWER:

a) was I able / moose and reindeer

The presence of back then clearly indicates that a past tense is needed. It is, therefore, possible to rule out b) am I able and d) am I being able immediately. Since the Past Simple and Past Perfect Tense in this case describe an event in the distant past, either is appropriate here. As far as the second blank is concerned, the missing item is a noun phrase in the plural. The nouns moose and reindeer belong to the category of nouns which have the same form in the singular and plural, so options (a) and (d) are correct, yet only (a) appropriately completes both the first and second blank. POINTS OF INTEREST: tenses, nouns with the same form for sing./pl.

12. CORRECT ANSWER:

c) dress / cast / loosen up

The first blank requires the use of a word which collocates with rehearsal. b) clothes and d) general can be eliminated right away, while c) dress rehearsal is ‘the final rehearsal of a play, opera or show’, so either a) final or c) dress is appropriate here. Further, the second blank requires a noun denoting ‘the people who act’ in Macbeth. Potential correct choices include b) party meaning ‘a group of people who are doing something together’, c) cast ‘all the performers in a play or film’ and d) troupe ‘a group of dancers, singers and actors who travel to different places to perform’. a) crew is inappropriate since it is used to denote ‘people who work on a ship or an aircraft’. Finally, when it comes to the third blank, it should be completed with a suitable phrasal verb. Of the four solutions, only c) loosen up, in the sense ‘become relaxed and less tense’ and b) get together, meaning ‘meet in order to spend time together’ meaningfully complete the blank. The other two options a) lose out, ‘suffer a loss or disadvantage’, and d) get lost, ‘a rude way to tell someone to go away’, do not make sense in this context. POINTS OF INTEREST: collocations, phrasal verbs page | 39

Mastering Proficiency Skills in English

13. We completely understand your arguments. You have __________ and we are definitely going to __________ consideration during the decision making. a) scored a point / take them on b) come to the point / take them in c) made your point / take them into d) pointed them / take them onto

14. __________ and robbed twice in the Italian __________, Joshua decided that he __________ there anymore. a) Attacked / block / won’t go b) Having attacking / district / couldn’t go c) Having attacked / section / shouldn’t go d) Having been attacked / quarter / wouldn’t go

15. The government ought to pass a new law if the victims __________ for their loss. a) are to compensate b) will compensate c) are to be compensated d) will be compensating

page | 40

Grammar/Vocabulary

13. CORRECT ANSWER:

c) made your point / take them into

Both b) come to the point and c) made your point are appropriate for the first blank, the former meaning ‘start talking about something that is important to someone’ and the latter ‘proving that something is true’. Options (a) and (d) do not make sense in this context. As for the second blank, the fixed expression is to take something into consideration and therefore, only answer (c) is correct. Other options do not fit the context: a) take them on most commonly denotes ‘to begin to perform or deal with’ i.e. new responsibilities/an opponent, b) take them in ‘to receive someone as a guest or lodger/to perceive, understand something’, whereas d) take them onto is an Englishlike nonexpression. POINTS OF INTEREST: idioms, verb + preposition

14. CORRECT ANSWER:

d) Having been attacked / quarter / he wouldn’t go

The first blank requires a PARTICIPLE form which will clearly indicate the order in which the events described in the sentence occurred. Attack is a transitive verb, yet no object is provided in this sentence, which means that Joshua was a victim, not a perpetrator, so c) having attacked can be disqualified right away. b) Having attacking is inappropriate since this form is non-existent leaving c) attacked, the past participle form, and d) having been attacked, the perfect participle passive, as possible correct choices. The second blank needs to be completed with a suitable noun meaning ‘a particular area of a town’. Consequently, it is immediately possible to rule out a) block and c) section which do not make sense in this context while both b) district and d) quarter are appropriate here. Finally, the choice of a verb for the third blank is governed by the rule of the Sequence of Tenses as the verb decided is the Past Simple Tense form. Therefore, only item a) won’t go does not fit in, yet only (d) meaningfully completes all blanks. POINTS OF INTEREST: participles, Sequence of Tenses

15. CORRECT ANSWER:

c) are to be compensated

An appropriate verb form is needed to complete a conditional sentence. The choice of the correct answer is governed by the fact that the intended meaning of this sentence is that the victims should get compensation. Therefore, all options but c) are to be compensated, the passive structure, should be ruled out. This kind of structure be to + INFINITIVE, is common in ifCLAUSES when the main clause expresses a pre-condition – something that must happen first if something else is to happen. POINTS OF INTEREST: passive voice

page | 41

Mastering Proficiency Skills in English

16. It is essential that this thorny problem __________ as soon as possible by experts __________ dieting. a) tackles / of b) should tackle / in c) be tackled / on d) should be tackled / for

17. It’s worth __________ that people make mistakes because we are all only human, __________. a) to point out / made of flesh and blood b) pointing out / made of flash and blood c) to note / made of flash and blood d) noting / made of flesh and blood

18. Don’t worry, I’ll __________ you what you haven’t __________ in school, as a student. a) teach / learned b) learn / thought c) learn / studied d) teach / taught

page | 42

Grammar/Vocabulary

16. CORRECT ANSWER:

c) be tackled / on

It is essential is a structure which is commonly followed by either should + BARE INFINITIVE or the PRESENT SUBJUNCTIVE (identical in form to the BARE INFINITIVE). What is more, the presence of …by experts clearly indicates that what is missing in the first blank is a passive verb form so it is immediately possible to rule out a) tackles and b) should tackle. As for the second blank, the preposition which collocates with the noun expert is c) on. POINTS OF INTEREST: subjunctive, noun + preposition

17. CORRECT ANSWER:

d) noting / made of flesh and blood

The adjective worth is followed by GERUND, not INFINITIVE so items a) to point out and c) to note can be eliminated right away. The other two options b) pointing out and d) noting are potential correct answers whose meaning is almost synonymous, ‘drawing attention to something’. The choice of the right answer, therefore, depends on the second blank. Since the collocation is ‘flesh and blood’ in the sense ‘having human feelings or weaknesses’, (a) and (d) are acceptable options, yet only (d) fits both blanks. POINTS OF INTEREST: gerund, collocations

18. CORRECT ANSWER:

a) teach / learned

Both blanks require the use of an appropriate verb: either to teach or to learn. The first blank is followed by an object you so it is clear that the verb offered in a) and d) teach is needed here. (*learn you is an impossible combination) As for the second blank, since we learn things at school as students, the best option is a) learned. The other items are inadequate due to their meaning: b) thought is the Past Simple Tense form of the verb to think, c) studied means ‘to engage in a learning process (usually at home, not in school)’ or ‘to analyze attentively in detail in order to research something’, while d) taught is the Past Simple Tense form of the verb to teach.

page | 43

Mastering Proficiency Skills in English

19. The __________ mountain scenery __________ every tourist that sees it. a) bear / struck b) barren / struck c) barely / strikes d) bare / strikes

20. The acoustics of this hall __________ amazing. __________, only __________ musicians have held concerts in it. a) are / However / few b) is / Still / a few c) are / Yet / a few d) is / Nevertheless / few

21. My new employer inquired __________ any other language __________ French and Japanese. a) if I spoke / next to b) did I speak / in addition to c) if I spoke / besides d) did I speak / beside

page | 44

Grammar/Vocabulary

19. CORRECT ANSWER:

d) bare / strikes

The first blank requires an appropriate adjective. Since a) bear is a noun and c) barely is an adverb, these must be ruled out. Both b) barren and d) bare are adjectives, the former meaning ‘dry and bare’ and the latter ‘having no plants growing on it’, so the choice of the correct answer depends on the second blank and the form of the verb to strike (strike-struck-struck). Judging from the context, e.g. every tourist that sees it, it is clear that the tense used in this sentence is the Present Simple. In order to meaningfully complete the second blank d) strikes must be selected.

20. CORRECT ANSWER:

d) are / However / few

Firstly, the noun acoustics is plural in form and it takes a plural verb here, meaning ‘the way the sound is shaped in a room’. Therefore, b) and d) is can be eliminated right away. Secondly, any one of the four options offered for the second blank is a possible solution, since all of them introduce a statement contradictory to the one previously mentioned. Finally, the item missing in the third blank is an appropriate adjective expressing a contradiction between the acoustic quality of the concert hall and the number of musicians who have held concerts there. While b) and c) a few has a positive meaning which is inappropriate in this context, a) and d) few possesses a negative meaning ‘not as many as one would like/expect’. (d) is, therefore, the best choice for all blanks. POINTS OF INTEREST: plural nouns, adverbs, a few/few

21. CORRECT ANSWER:

c) if I spoke / besides

The choice of the potential correct answers for the first blank is governed by the rules of reported speech due to the presence of the Past Simple Tense form inquired. Since an interrogative form is presented in b) and d) did I speak, these two options must be ruled out. Moreover, the second blank should be completed with a preposition meaning ‘apart from’. Both b) in addition to and c) besides have this meaning, but only (c) meaningfully completes both blanks. Other options, a) next to and d) beside, are inappropriate here as their meaning is ‘at the side of’ e.g. Who’s that blonde sitting next to/beside Peter? POINTS OF INTEREST: reported speech, confusable words

page | 45

Mastering Proficiency Skills in English

22. __________ you happen to see Alice, don’t forget to __________ me to her, __________? a) Would / remind / won’t you b) Whether / remember / will you c) If / remind / won’t you d) Should / remember / will you

23. Sally __________ next week. __________ her right for drunken driving! a) will revoke her driver’s license / Deserves b) will have revoked her driver’s license / Does c) will have her driver’s license revoked / Serves d) will have had her driver’s license revoked / Makes

24. Only America __________ the world in any __________ way. a) effects / meaningless b) expects / mean c) infects / meaning d) affects / meaningful

page | 46

Grammar/Vocabulary

22. CORRECT ANSWER:

d) Should / remember / will you

There are two possible answers for the first blank. Both c) if and d) should, which corresponds to ‘If you should happen…’, denote that something is unlikely, or not particularly probable. The other two options a) would and b) whether do not make sense in this context. Furthermore, the second blank demands a verb used when asking someone to pass your greetings on to somebody else, that is b) and d) remember. Finally, a question tag is missing in the third blank. Since the question tag follows a negative imperative don’t forget, only b) and d) will you is appropriate here so (d) is the best solution to all three blanks. POINTS OF INTEREST: conditional sentences, question tags

23. CORRECT ANSWER:

c) will have her driver’s license revoked / Serves

The intended meaning of the first sentence is that people in authority will cancel Sally’s driver’s license due to a punishable offense - drunken driving. have + OBJECT + PAST PARTICIPLE construction can be used when the subject is the person who suffers as a result of the action. Therefore, items (a) and (b), namely, the Future and Future Perfect Tense, which suggest that Sally will be the one who is canceling her driver’s license, can be disqualified right away. Since the time reference is next week and only one action is mentioned in this sentence, the best choice is (c), not (d) which implies that an action will be completed before some other action in the future. As for the second blank, the expression used when something unpleasant is about to happen to someone and no one shows any sympathy for them because it is believed that it was their own fault is serves someone right. POINTS OF INTEREST: causative have, collocations

24. CORRECT ANSWER:

d) affects / meaningful

The first blank requires a suitable transitive verb meaning ‘change or influence in some way’, that is d) affects. Option b) expects can be eliminated immediately as it doesn’t make sense in this context. Items a) effects and c) infects are also inappropriate here since their meanings are ‘succeed in achieving something’ and ‘contaminate with harmful germs or bacteria’, respectively. The second blank should be filled in with an adjective. While c) meaning is a noun, b) mean is also not a possibility as it describes someone who is stingy or unkind. Only d) meaningful can be used here, an adjective consisting of meaning + -ful, denoting ‘having meaning’. Item a) meaningless is an antonym of meaningful, denoting ‘without meaning’ (compare hopeful-hopeless, careful-careless) and, therefore, does not make sense in this context. POINTS OF INTEREST: confusable words, word formation by means of suffixes page | 47

Mastering Proficiency Skills in English

25. __________ of time we __________ the deadline and handed in the paper. a) At the end / missed b) In the nick / met c) At the top / missed d) In the moment / met

26. The management team is very fond of him. He’s been __________ since he started working for the corporation. a) in their bad books b) in their good books c) reading their books d) reading them like books

27. Underground I work __________ a slave yet get paid __________ a typical coal miner. a) like / as b) like / like c) as / as d) as / like

page | 48

Grammar/Vocabulary

25. CORRECT ANSWER:

b) In the nick / met

The first blank should be completed with an appropriate expression which collocates with … of time. The only correct option is b) in the nick as it is a part of a phrase in the nick of time meaning ‘at the last possible moment’. Moreover, since the second part of the sentence states that the paper was handed in, it becomes clear that the deadline was b) or d) met, not a) or c) missed.

26. CORRECT ANSWER:

b) in their good books

Only (b) is appropriate in this context as to be in someone’s good books means that someone is pleased with you. The meaning of a) in their bad books is quite the opposite, that is, that someone is annoyed with you, while c) reading their books and d) reading them like books are not English idiomatic expressions at all.

27. CORRECT ANSWER:

a) like / as

The choice of the correct answers for both blanks is governed by the fact that the aforementioned person is not really a slave, but a coal miner, who is complaining about his hard work by comparing it to a slave’s. The preposition as is used before nouns to say what jobs people do, not like, e.g. If you are a professor, you could say I work as a professor at the faculty.

page | 49

Mastering Proficiency Skills in English

28. He tore off a __________ of bread and __________ it on her palm. a) slice / put b) hunk / placed c) crumb / gave d) piece / passed

29. Mass __________ informs us __________ the latest local and international news. a)

media / of

b) medium / regarding c)

media / about

d) medium / on

30. __________ that I decided not to read it till the end. That’s how I missed valuable __________ about the changes in policy affecting maternity __________. a) So boring was the article / pieces of information / leave b) So bored the article was / information / absence c) So boring the article was / pieces of informations / break d) So bored was the article / piece of information / period

page | 50

Grammar/Vocabulary

28. CORRECT ANSWER:

b) hunk / placed

The noun missing in the first blank should meaningfully collocate with the verb tore off. Therefore, a) slice which denotes ‘a thin piece that has been cut’ can be ruled out. c) crumb, meaning ‘tiny piece that falls from bread’, can also be ruled out. b) hunk and d) piece are equally appropriate here. For the second blank a) put and b) placed are potential good choices due to their meaning. c) gave and d) passed do not fit in with the rest of the sentence …it on her palm. However, only (b) is well-suited for both blanks.

29. CORRECT ANSWER:

a) media / about

The correct expression is mass media meaning ‘television, radio, newspapers, and magazines which give information and news to large numbers of people’. The meaning of the noun medium is ‘a way of expressing your ideas/substance or material which is used for a particular purpose’. Further, the verb inform is commonly followed by the preposition of or a that-CLAUSE, so all options offered apart from (a) are inappropriate for the second blank.

30. CORRECT ANSWER: leave

a) So boring was the article / pieces of information /

The first blank requires an adjective with the meaning ‘having this effect’, i.e. a PRESENT PARTICIPLE functioning as an adjective, not an adjective with the meaning ‘affected in this way’, i.e. a PAST PARTICIPLE. It is used here in an emphatic structure so + ADJECTIVE + VERB + NOUN, which requires inversion. Therefore, (b) and (d) can be disqualified right away due to the inappropriate participle. (c) has to be ruled out because the word order in this item is not inverted. As for the second blank, information is an uncountable noun which can be made plural by the use of ‘a piece of…’ before it. Therefore, a) pieces of information and b) information are the only potential correct answers. Finally, the collocation missing in the third blank is maternity leave ‘a period of time when a woman is not working at her job because she is pregnant or has given birth’. POINTS OF INTEREST: inversion, uncountable nouns, collocations

page | 51

Mastering Proficiency Skills in English

TEST 3 1. I was shocked to see that the sisters looked exactly __________. a) b) c) d)

the like same alike like

2. __________ exams you fail, __________ your chances are of getting the scholarship. a) b) c) d)

Less / better The less / the better Fewer / better The fewer / the better

3. The __________ to the cottage completely __________ us. a) b) c) d)

page | 52

three days’ journey / exhausted three day journey / exhausting three-days’ journey / exhausts three day’s journey / exhaust

Grammar/Vocabulary

1. CORRECT ANSWER:

c) alike

Being an adjective alike may follow a LINK VERB, or COPULA (such as the verb to look in the sense ‘to seem, to appear’, the sense in which the verb is used in this sentence). Further, alike is one of a group of adjectives beginning with a- which generally appear only following a linking verb, that is, in predicative position (e.g. awake, asleep, alive, alone, afraid). a) the like, although a noun that may appear in a predicative position as well, is not the best choice because of its meaning - ‘a counterpart or corresponding item/person’. b) same is another adjective offered, but same is used with the or another demonstrative and none is provided. d) like, functioning as an adjective, does not appear in predicative position.

2. CORRECT ANSWER:

d) The fewer / the better

Comparative forms of adjectives (such as fewer and better) are used here with the definite article to indicate that the two changes are seen as complementary. b) The less / the better, which also makes use of comparatives with the, is not the best choice because the adjective few is used with plural nouns (such as exams), whereas the adjective little is used with singular (usually uncountable) nouns. a) Less / better and c) Fewer / better omit the definite article and are, therefore, not appropriate here.

3. CORRECT ANSWER:

a) three days’ journey / exhausted

The first blank in the sentence requires a time expression. One possibility is to use the -s genitive, a) three days’ journey, while the other is to use the ‘noun as adjective’ structure, e.g. three-day journey, with an obligatory hyphen. Therefore, b) three day journey is inappropriate. d) three day’s journey is inadequate because the noun day is not in the plural. c) three-days’ journey is inadequate because the ‘noun as adjective’ structure does not contain a noun in the plural, nor does it have a possessive form (‘). The second blank requires a verb form. It is possible to immediately rule out b) exhausting and d) exhaust because a FINITE VERB form is needed; further, if d) exhaust were considered the Present Simple Tense form, it would, once again, not be appropriate because it does not agree with the subject, which is in the third person singular. Either a) exhausted or c) exhausts would be appropriate for the second blank but only answer (a) appropriately completes both the first and the second blank. POINTS OF INTEREST: possessive case, tenses page | 53

Mastering Proficiency Skills in English

4. We can conclude that __________ of their children __________ a good education. a) b) c) d)

every / have had all / has had each / has had no one / have had

5. Upon asking our parents who they wanted __________ to their celebration, my sister Alice and __________ wrote the invitations. a) b) c) d)

to call / me to invite / I to request / I to bring / myself

6. For his birthday, the boy received a __________ baseball bat and a __________ ball. a) b) c) d)

page | 54

wooden blue new / leather soft blue wooden new / soft leather new blue wooden / soft leather blue new wooden / leather soft

Grammar/Vocabulary

4. CORRECT ANSWER:

c) each / has had

While both b) all and c) each may occur followed by of, neither a) every nor d) no one may be followed by of. The second blank requires a verb form which agrees with the subject. If the subject selected is all of their children, then the subject should be in the plural. However, the verb form offered in (b) is in the singular. If the selected subject is each of their children, the subject should be in the singular, as is the case in (c). POINTS OF INTEREST: each/every/all, subject-verb agreement

5. CORRECT ANSWER:

b) to invite / I

The choice of the verb in the first blank is governed by the need for it to collocate with celebration. a) to call and c) to request, therefore, would not be appropriate here, while either (b) or (d) could be suitable. The second blank requires a pronoun in the COMMON (NOMINATIVE) CASE as this is part of the subject of the verb wrote. Therefore, neither a) me nor d) myself is appropriate. The first is in the OBJECT (ACCUSATIVE) CASE, whereas the second is a REFLEXIVE PRONOUN. Only (b), however, gives appropriate solutions to both blanks.

6. CORRECT ANSWER:

c) new blue wooden / soft leather

Both blanks require adjectives in ATTRIBUTIVE POSITION, that is, preceding the noun. The adjectives offered are adjectives of quality and these follow a fairly standard order: size (except little)

general age and shape description little

color

material

origin

purpose

1) size (except little), 2) general description (excluding adjectives of personality, emotion, etc.), 3) age and little, 4) shape, 5) color, 6) material, 7) origin, 8) purpose (these are generally gerunds used to form compound nouns). The adjective new gives a general description, blue indicates color, wooden material, while soft gives a general description and leather designates material. POINTS OF INTEREST: order of adjectives

page | 55

Mastering Proficiency Skills in English

7. Standing __________ the doorway, her arms __________, she sized me up. a) b) c) d)

on / crossed in / folded at / clapped over / held

8. After __________ months of work he had persuaded 70 per cent of the workers __________ the contract in the end. a) b) c) d)

few / to sign several / to sign any / to applicate a few / to applicate

9. The outing was canceled and no one __________ her sister, who wasn’t __________, came. a) b) c) d)

page | 56

apart / out-of-date beside / updated apart from / up-to-date besides / outdated

Grammar/Vocabulary

7. CORRECT ANSWER:

b) in / folded.

The preposition in is used to describe a position in a place that has three dimensions – length, breadth and depth – such as a doorway. The preposition a) on is used to describe a position on a line or on a surface, c) at is used to describe a position at a point, while d) over is used to mean ‘higher than’ or ‘covering, crossing’. The second blank requires a PAST PARTICIPLE to describe the position of the arms; a) crossed and b) folded would be appropriate, while c) clapped and d) held would not because the meaning of these verbs does not allow the collocation with arms to describe the positioning of these extremities.

8. CORRECT ANSWER:

b) several / to sign

The first blank is followed by a noun in the plural, months, and from the context it is possible to conclude that the intended meaning is ‘an unspecified number of months’. Therefore, either b) several or d) a few are possible solutions to the first blank. a) few possesses a negative meaning ‘not as many as one would like/expect’, and is therefore unsuitable here, while c) any is not appropriate here because some is commonly used in affirmative sentences to denote the abovementioned meaning. The second blank requires an appropriate verb. The solution offered in c) and d) to applicate is an Englishlike nonword. Instead, to sign, offered in (a) and (b) is correct, yet only (b) is a well-suited solution to both blanks.

9. CORRECT ANSWER:

c) apart from / up-to-date

The first blank requires a preposition meaning ‘except’. Only c) apart from is used specifically to ‘exclude something or someone’, and is adequate here. The other three options are inappropriate: a) apart can be used as an adjective or an adverb, but not as a preposition; b) beside is used to mean ‘at the side of’ or ‘by’; finally, as a preposition, d) besides is used to mean ‘in addition to’ or ‘ as well as’. The second blank requires an adjective. While a) out-of-date, c) up-to-date and d) outdated are all adjectives, b) updated is the past participle of the transitive verb to update. Of the three adjectives offered, only c) up-to-date in the sense of ‘abreast of the information that the outing was canceled’ meaningfully completes the blank. The meaning of d) outdated and a) out-of-date, ‘outmoded, obsolete, no longer current’, is not appropriate here. POINTS OF INTEREST: prepositions, compound adjectives page | 57

Mastering Proficiency Skills in English

10. I do hope you haven’t had __________ day at work. I would like us to lend Ann, __________ is redecorating her apartment, a helping hand. a) b) c) d)

a too exhausting / whose a too exhausted / which too exhausted a / that too exhausting a / who

11. __________ of my appearance people see me as a threat and they try to inflict __________ harm upon me. a) b) c) d)

In spite / body Despite / physically Due / physical Because / bodily

12. He was __________ before __________ finish giving me an update over the phone. a) b) c) d)

page | 58

cut off / being able to cut out / he was able to cut out / he had been able to cut off / having been able to

Grammar/Vocabulary

10. CORRECT ANSWER:

d) too exhausting a / who

The first blank requires an adjective with the meaning ‘having this effect’, i.e. a PRESENT PARTICIPLE functioning as an adjective, not an adjective with the meaning ‘affected in this way’, i.e. a PAST PARTICIPLE. Therefore both b) a too exhausted and c) too exhausted can be ruled out. Further, when the adverb of degree too modifies an adjective which precedes a noun, the indefinite article a/an occurs before the noun, rather than in front of the adjective modified by too: too exhausting a not *a too exhausting. The second blank requires completion of a RELATIVE CLAUSE. What is missing is a RELATIVE PRONOUN which is the subject of the relative clause and refers to the noun Ann so it is possible to rule out b) which, as the relative pronoun which is not used to refer to persons. Moreover, in this kind of relative clause (which provides additional information and is separated from its noun referent by commas) where the relative pronoun refers to a person, it is possible to use only d) who, never that, which excludes c) that. Finally, a) whose is a possessive and needs to be followed by a noun, which is not the case here. POINTS OF INTEREST: adjectives ending in –ing/–ed, relative pronouns

11. CORRECT ANSWER:

d) Because / bodily

As the first blank is followed by OF + NOUN PHRASE (my appearance), it is necessary to choose a preposition that ends in of. Either a) in spite or d) because would be possible, as in spite of and because of are prepositions. Neither c) due nor b) despite are appropriate here as the preposition is due to, not *due of, and despite is a single-word preposition (*despite of). The second blank requires an adjective to modify the noun harm. It is immediately possible to rule out both a) body and b) physically, as the first is a noun and the second an adverb (remember that both adverbs and adjectives, e.g. friendly, may end in –ly). Either c) physical or d) bodily are appropriate, but as due is not appropriate for the first blank, only answer (d) correctly completes the sentence. POINTS OF INTEREST: prepositions, adjectives

12. CORRECT ANSWER:

a) cut off / being able to

Two phrasal verbs are offered for the first blank: cut off, a transitive verb meaning ‘disconnect’, and cut out, an intransitive verb meaning ‘be inconveniently isolated’; only cut off is appropriate for this sentence. The possible solution to the second blank is the Simple Past Tense, as in b) he was able to but not the Past Perfect Tense, as in c) he had been able to, because it implies that an action was completed before another one started, which is not the case in this sentence (first the person was cut off so that is why he couldn’t finish giving an update). Furthermore, another possible option is the SIMPLE GERUND, a) being able to, because the blank is preceded by a preposition, but not the PERFECT GERUND, d) having been able to, because it is used in the same manner as the Past Perfect Tense - to emphasize that an action is complete before another one starts. Only answer (a), however, meaningfully completes both blanks. POINTS OF INTEREST: phrasal verbs, gerund page | 59

Mastering Proficiency Skills in English

13. The __________ accounts of the accident coincide with the evidence the investigators have found so far. They are still __________ the matter, though. a) b) c) d)

passer-by’s / looking for passer’s-by / looking out for passers-by’ / looking up to passers’-by / looking into

14. __________ the title Honorary Professor at __________ Newcastle University, Christian moved to Newcastle. a) b) c) d)

Awarding / the Having awarded / Having been awarded / Been awarded / the

15. We will never know __________ the bottom drawer of the antique desk __________ as a hiding place for the valuable possessions in the nineteenth century. a) b) c) d)

page | 60

weather / could have served whether / has served whether / might have served weather / had served

Grammar/Vocabulary

13. CORRECT ANSWER:

d) passers’-by / looking into

The first blank requires a POSSESSIVE (GENITIVE) FORM of the noun passer-by in the plural (note …accounts). This is a COMPOUND NOUN formed from a noun ending in -er, passer, and an adverb, by. This type of a compound noun does not form the plural by adding –s at the end of the noun, but by adding it to the noun part of the compound (passer) instead. An apostrophe (’) is then added after it to indicate the possessive case in the spelling. Of the four phrasal verbs offered to fill in the second blank, d) looking into meaning ‘investigate’ best fits the context of the sentence.

14. CORRECT ANSWER:

c) Having been awarded / -

The first blank requires a participle form which will clearly indicate the order in which the events described in the sentence occurred. PERFECT PARTICIPLES are used to emphasize that an action has been completed before the beginning of another one. It is possible to conclude from the context that Christian was awarded the title Honorary Professor before his move to Newcastle. In this case the PERFECT PARTICIPLE, b) having awarded or c) having been awarded is needed. As Christian is the recipient of the title, not the person giving an award to someone, (b), the PERFECT PARTICIPLE ACTIVE, has to be ruled out. Further, d) been awarded is not a correct verbal construction, while a) awarding, the PRESENT PARTICIPLE, is not appropriate here too because it would have to be followed by the preposition TO (note award something TO someone). The second blank requires an article; in (b) and (c) the zero article is offered, while in (a) and (d) there is the definite article. Names of universities typically follow one of two patterns: - when a person’s name or a place name precedes the noun university (e.g. ‘Monash University’, ‘Edith Cowan University’) we use zero article. - when the noun university is followed by of and a place name (e.g. ‘the University of Melbourne’, ‘the University of Queensland’) we use the definite article. Therefore, zero article should be used before Newcastle University. POINTS OF INTEREST: participles, definite article

15. CORRECT ANSWER:

c) whether / might have served

To complete the first blank, a choice must be made between two homophones (words which sound the same but are spelled differently and have different meanings): weather ‘atmospheric conditions’ and whether, a conjunction used to introduce an indirect question involving stated or implied alternatives. Once it is clear that the conjunction whether completes the first blank, it should also be clear that the indirect question which follows involves alternatives – either the bottom drawer served as a hiding place or it did not serve as a hiding place. The most logical choice, therefore, is c) might have served as may/might can be used to imply that both what is stated and its alternative (even if unstated) are possible. Further, a) could have served is another possibility, as the meaning of can/could is close to the meaning of may/might, with the exception, most notably, of the use of can/could to talk about ability; b) has served and d) had served are inappropriate because the Present Perfect Tense cannot be used to talk about the distant past with no consequences in the present (note …in the nineteenth century), that is, because there is no other past tense in the sentence and, therefore, no justification for the use of the Past Perfect Tense. POINTS OF INTEREST: modals in the past page | 61

Mastering Proficiency Skills in English

16. __________ in politics one has to master the __________ of public speaking, be __________ and listen to the voice of the ordinary people. a) b) c) d)

To succeed / ability / likely Succeeding / craft / liking To succeed / art / likeable Succeeding / skill / liked

17. With new evaluation techniques constantly __________, soon teachers __________ gain more insight into their pupils’ __________. a) b) c) d)

improving / are able to / strengths and weaknesses improved / can / strength and weakness being improved / will be able to / strengths and weaknesses having been improved / could / strength and weakness

18. By the end of this year Stephen __________ here for twelve years. His parents are very disappointed in him, and believe he __________ medicine as his future profession. a) b) c) d)

page | 62

will be studying / shouldn’t choose is to study / ought not to have chosen is going to study / ought not to choose will have been studying / shouldn’t have chosen

Grammar/Vocabulary

16. CORRECT ANSWER:

c) To succeed / art / likeable

The first blank requires an appropriate form of the verb succeed. As the FULL INFINITIVE (with to) can be used to indicate purpose (‘in order to succeed’), both (a) and (c) are appropriate for the first blank. The ing-form in (b) and (d) succeeding cannot be used to indicate purpose in this way. The second blank requires a noun with the appropriate meaning; public speaking is often described as an c) art, but b) craft and d) skill are potential good answers too. Only a) ability is inappropriate since this noun cannot be followed by of. The third blank requires an adjective. It is possible to immediately rule out b) liking, as it is either a noun or, as the present participle of the verb to like, is not used as an adjective. a) likely, in the sense of ‘attractive’, although possible, has to be ruled out by the unsuitable choice for the second blank. c) likeable meaning ‘having qualities that bring about a favorable regard’ is a good potential choice for blank three. Finally, even though d) liked is initially a possible choice as part of the PASSIVE INFINITIVE of the verb to like, i.e. be liked, answer (d) must be ruled out because it does not offer the full infinitive for the first blank in the sentence. Therefore, only (c) meaningfully completes all three blanks. POINTS OF INTEREST: infinitive, word formation by means of suffixes

17. CORRECT ANSWER: weaknesses

c) being improved / will be able to / strengths and

The first blank is part of a SUBORDINATE CLAUSE beginning with with and expressing accompanying circumstances. The blank is preceded by constantly, which requires a PROGRESSIVE PARTICIPLE. It is therefore possible to rule out b) improved, the PAST PARTICIPLE, and d) having been improved, the PERFECT PARTICIPLE PASSIVE, immediately. The second blank requires a verb form expressing future because it is preceded by the adverb soon. a) are able to can be ruled out as the verb is in the Simple Present Tense; b) can should also be excluded as it is not used to talk about a future possibility. Either c) will be able to, used to indicate a future ability, or d) could, used to indicate a future possibility, could be used to fill in this blank. Finally, the third blank requires the plural form (note plural pupils’) of the two nouns joined by and, as in (a) and (c). POINTS OF INTEREST: participles, modal verb can

18. CORRECT ANSWER:

d) will have been studying / shouldn’t have chosen

The presence of by + time expression (by the end of this year) clearly indicates that Stephen’s studying, a long-lasting action, will continue up to a future point in time. Only the Future Perfect Continuous (d) is therefore appropriate for the first blank. The second blank has to be considered in relation to the fact that Stephen’s parents are disappointed in him and the intended meaning of the sentence is that Stephen made a wrong/foolish decision in the past, that is, he d) shouldn’t have chosen or b) ought not to have chosen medicine as his future profession. a) shouldn’t choose and c) ought not to choose do not express this past time reference. Only (d), however, is the best choice for both blanks. POINTS OF INTEREST: future tenses, modal verbs page | 63

Mastering Proficiency Skills in English

19. The farmer is extremely satisfied as the cattle stolen from his ranch ______. a) b) c) d)

is returned has been returned are returned have been returned

20. It was proved that the mineral content of human bones __________ with age, __________ them increasingly fragile. a) b) c) d)

has decreased / made decreased / made decreased / making has decreased / making

21. Had any of us __________ in the incident at work last week, I am sure we __________ in danger of losing our jobs then and there. a) b) c) d)

page | 64

had some involvement / would be had any involvement / would have been involved / would have been been involved / would be

Grammar/Vocabulary

19. CORRECT ANSWER:

d) have been returned

Firstly, the noun cattle is one of several nouns which are plural but do not end in –s. A plural form of the verb is therefore needed, so either c) are returned or d) have been returned might be possible. Secondly, as the action being described is a past action with no definite time reference but with results in the present (note is … satisfied), the Present Perfect Tense, not the Simple Present Tense is appropriate here. POINTS OF INTEREST: plural nouns

20. CORRECT ANSWER:

c) decreased / making

The verb in the main clause, was proved, is in the Simple Past Tense and whenever this is the case, verbs in subordinate clauses undergo the Sequence of Tenses. This immediately rules out a) and d) has decreased. The second blank, also part of a subordinate clause, needs to be completed with either the PRESENT PARTICIPLE or the PAST PARTICIPLE of make. As the second action is a result of the first, it can be expressed by the present participle which has the ability to shorten the sentence (note the use of the comma). Therefore the past participle in (a) and (b) is an ungrammatical option and the choice of the correct answer is narrowed down to the present participle in c) and d) making, yet only (c) meaningfully completes both blanks. POINTS OF INTEREST: Sequence of Tenses, participles

21. CORRECT ANSWER:

b) had any involvement / would have been

The presence of …then and there makes is clear that this is a CONDITIONAL SENTENCE of the so-called third type where Had any of us… is used instead of If any of us had…The tenses typically used in this type are IF + PAST PERFECT,… WOULD + PERFECT INFINITIVE, as is the case in (b). All the other options are ungrammatical in this context. POINTS OF INTEREST: conditional sentences

page | 65

Mastering Proficiency Skills in English

22. Eleanor Roosevelt set the standard against which the wives of all United States Presidents __________ ever since. a) b) c) d)

are evaluating have evaluated have been evaluated had been evaluating

23. Oh my God! I __________ at the laundry and now __________ that I can’t __________. a) b) c) d)

washed my pants / it’s too baggy / get into them had had my pants washed / they’re too baggy / fit into them have washed my pants / it’s so tight / put them on had my pants washed / they’re so tight / wear them

24. When the new class president was elected we all __________ and hoped that he would live up to his promises. a) b) c) d)

page | 66

eyed him up and down cast an eye gave him an eye caught his eyes

Grammar/Vocabulary

22. CORRECT ANSWER:

c) have been evaluated

The presence of ever since clearly indicates that the Present Perfect Tense is needed. Further, the choice of the verb evaluate indicates that the Present Perfect Passive is the best choice, as the wives of all United States Presidents since Eleanor Roosevelt was First Lady are the object of evaluation. POINTS OF INTEREST: passive voice

23. CORRECT ANSWER: them

d) had my pants washed / they’re so tight / wear

Three of the four options offered for the first blank are a possible solution: the Simple Past of wash a) washed is acceptable, as is the Present Perfect c) have washed. Also acceptable is the Simple Past of have in the structure have + OBJECT (my pants) + PAST PARTICIPLE (washed), meaning ‘employ someone to wash my pants’ (the so-called CAUSATIVE HAVE) as in d) had my pants washed. Solution b) had had my pants washed is clearly incorrect as there is no other past tense in the sentence and therefore no reason to use the Past Perfect Tense. In order to find the correct solution to the second blank, it is necessary to keep in mind that pants is a plural noun, which immediately disqualifies a) it’s too baggy and c) it’s so tight due to the use of an unsuitable pronoun, namely it. Further, the second blank ends in an adjective, which is then followed by that in the subordinate clause (that I can’t …). This indicates that what is required is so + ADJECTIVE + that, which rules out b) they’re too baggy. Any of the options offered for the third blank are possible solutions. POINTS OF INTEREST: causative have, plural nouns

24. CORRECT ANSWER:

a) eyed him up and down

The only suitable answer is a) eyed him up and down, in the sense of ‘studied him carefully’. The correct expression is not b) cast an eye, but cast a glance. The next option, c) gave him an eye, is also wrong; the correct expression is to give someone the eye. Finally, instead of d) caught his eyes, the correct expression would be caught his eye.

page | 67

Mastering Proficiency Skills in English

25. Henry completed five tasks in two minutes’ time. See if you can __________! a) b) c) d)

beat that beat it beat them beat around the bush

26. Making the beds is one of the __________ I don’t like __________. I’d rather do anything else. a) b) c) d)

activities / least of all duties / least jobs / the least chores / in the least

27. I’m __________ my best to solve this puzzle but it seems to be beyond my __________. a) b) c) d)

page | 68

working / capabilities doing / possibility giving / capacity doing / abilities

Grammar/Vocabulary

25. CORRECT ANSWER:

a) beat that

The only appropriate answer is a) beat that, in the sense ‘surpass Henry’s accomplishment’. ‘Go away; get out’ is the meaning of b) beat it; ‘to avoid answering a question; to stall; to waste time’ is the meaning of d) beat around the bush. Finally, c) beat them is inappropriate because of the object of the verb – them could be interpreted to refer only to five tasks, not to the fact that Henry completed five tasks in two minutes’ time.

26. CORRECT ANSWER:

d) chores / in the least

Any one of the three options offered for the first blank, excluding c) jobs as bed-making is not really a job, is a possibility, with a) activities being the least likely choice. However, only one of the options offered for the second blank is appropriate here. Considering that the verb in the subordinate clause (I don’t like …) is in the negative, it is possible to rule out a) least of all, b) least and c) the least as only (d) fits the negative context, indicating not liking something ‘the least bit, at all, even a little’.

27. CORRECT ANSWER:

d) doing / abilities

Only doing, as in (b) and (d) is a possible solution to the first blank. The other two options c) giving and a) working are not possible here as they don’t collocate with (my) best. As for the second blank, either a) capabilities or d) abilities would be possible, as these two words can be used interchangeably in many contexts, as well as c) capacity, if taken to denote the mental ability to understand, yet only (d) meaningfully completes both blanks.

page | 69

Mastering Proficiency Skills in English

28. There is a smoking section at the end of __________ hall because they don’t __________ smoking in the rest of the building. a) b) c) d)

each / permit every / forbid everyone / advise all / ban

29. Amy is much happier performing __________ than in recording __________. a) b) c) d)

alive / studios living / studioes live / studios lively / studioes

30. __________ of people gathered at the city square to __________ the Queen. a) b) c) d)

page | 70

Ten thousands / appraise Tens of thousand / salute Tens of thousands / greet Tens thousands / wave to

Grammar/Vocabulary

28. CORRECT ANSWER:

a) each / permit

Either a) each or b) every could be used to complete the first blank. Neither c) everyone nor d) all is a possible solution, the former because it is not a determiner, the latter because the noun hall is in the singular. For the second blank, a) permit is the most likely choice, although c) advise is possible but less likely. The remaining two options do not make sense in this context.

29. CORRECT ANSWER:

c) live / studios

The correct expression, which completes the first blank, is to perform live meaning ‘to perform during/from/at a live production’. In order to complete the second blank the correct spelling of the plural form of the noun studio must be selected. Unlike the plurals potatoes and tomatoes, the plural of studio is spelled studios (similar to photo-photos, piano-pianos, kilo-kilos, disco-discos). POINTS OF INTEREST: confusable words, plural of nouns

30. CORRECT ANSWER:

c) Tens of thousands / greet

Only the answer offered under (c) appropriately completes the first blank, as the only grammatically correct expressions are ten thousand and tens of thousands, the former not being one of the options. Any one of the options offered for the second blank could be used to complete the sentence. The verb appraise in (a), means ‘set a value on’, or ‘evaluate the worth, significance, merit of’; the verb salute in (b) is often used in the sense of ‘give a sign of respect, courtesy or goodwill to’; c) greet, similarly to salute, means ‘to address with expression of kind wishes upon meeting or arrival’, while d) wave to means to ‘move your hand from side to side in the air, usually in order to say hello or goodbye’.

page | 71

Mastering Proficiency Skills in English

TEST 4 1. Traveling in this vast land, we stumbled __________ an unusual hitchhiker, because of __________ our lives changed forever. a) over / which b) upon / whom c) at / who d) across / that

2. It’s the beginning of October and already __________ of the trees have lost __________ leaves. a) no / b) all / theirs c) none / their d) some / their

3. Pardon __________ you! Could you pass me that suitcase of __________? a) to be interrupting / my b) to interrupt / mine c) me interrupting / my d) me for interrupting / mine

page | 72

Grammar/Vocabulary

1. CORRECT ANSWER:

b) upon / whom

Only c) at does not collocate with the verb stumble, so the respective meanings of the three remaining combinations a) stumble over, b) stumble upon and d) stumble across have to be considered carefully: b) stumble upon and d) stumble across are identical ‘to find someone or something, usually by accident’. a) stumble over means ‘to trip in walking or running over (something/someone)’. Either b) stumble upon or d) stumble across, therefore, are the only options which fit the intended meaning of the sentence. Filling in the second blank completes a RELATIVE CLAUSE which does not describe a noun, but continues the narration. As d) that is not used in this type of relative clause, answer (d) can be ruled out. The second blank is preceded by because of, a preposition. As whom, but not who can appear as the object of a preposition, it is also possible to definitely rule out answer c) who. Finally, which, used here to refer to the entire preceding clause, is another pronoun which can be used to complete the sentence. However, answer (a) does not offer an appropriate solution to the first blank. This leaves (b) as the correct answer. POINTS OF INTEREST: phrasal verbs, relative pronouns

2. CORRECT ANSWER:

d) some / their

The first blank requires a pronoun; this immediately rules out a) no, which is a determiner, that is, comes directly before a noun group. Further, because of the presence of already, c) none does not make sense in this context, so b) all and d) some are the only possible solutions to the first blank. The second blank requires a determiner to precede the noun leaves. Zero article, as in a) – would be an appropriate solution here, as the noun is in the plural and does not represent any particular leaves. As it is not a determiner, but a pronoun, b) theirs has to be ruled out, which leaves a possessive adjective in c) and d) their as a potential correct answer. Together with some (d) gives the correct answer. POINTS OF INTEREST: no/none, possessive adjectives

3. CORRECT ANSWER:

d) me for interrupting / mine

The verb pardon, just like excuse and forgive, can be followed either by POSSESSIVE as in (c)) or by PRONOUN + PREPOSITION +

ADJECTIVE/PRONOUN + GERUND (e.g. my/me interrupting, GERUND (e.g. me for interrupting, as in (d)).

The second blank is preceded by the preposition of, therefore the correct solution has to be a pronoun which can be the object of the preposition. As my, in (a) and (c), is a possessive adjective, these two answers have to be ruled out. POINTS OF INTEREST: gerund, possessive adjectives/pronouns page | 73

Mastering Proficiency Skills in English

4. The small hotel was __________ visited by lovers who couldn’t afford a __________ rendezvous. a) mostly / smarter b) almost / cheaper c) most / more expensive d) barely / poshy

5. What __________ great news Susie has brought! This time next week we __________ in Nice, all __________ paid by our company. a) a / will stay / costs b) - / will be staying / expenses c) a / are going to stay / prices d) - / are staying / fees

6. He’d better not go out in this pouring rain, __________? a) would he b) had he c) wouldn’t he d) hadn’t he

page | 74

Grammar/Vocabulary

4. CORRECT ANSWER:

a) mostly / smarter

It follows from the intended meaning of the sentence that the first blank requires an adverb meaning ‘mainly’. Therefore, c) most can immediately be ruled out. The other two adverbs b) almost and d) barely do not have the intended meaning, so only a) mostly is an appropriate choice. The second blank requires an adjective to describe the noun rendezvous ‘date’. Only d) poshy can be immediately ruled out, as this is an Englishlike nonword (compare posh). The choice b) cheaper is unlikely as it does not make much sense in the context of this sentence. Finally, in addition to c) more expensive, a) smarter is a likely choice, as the adjective smart has not only the meanings ‘clever’ and ‘bright, knowledgeable’, but also the meaning ‘appealing to sophisticated tastes’. However, only answer (a) provides the correct solution to both blanks.

5. CORRECT ANSWER:

b) - / will be staying / expenses

When what is used in exclamations with singular countable nouns, the indefinite article must precede the noun. As news is an uncountable noun, the indefinite article is not required. Hence it is possible to rule out both a) and c) a. The second blank is preceded by this time next week and the context clearly indicates that the verb should express an action that will be in progress at a particular time in the future, which means that the Future Continuous Tense, b) will be staying, is needed. Finally, the correct expression, which completes the third blank, is all expenses paid. Choosing any one of the three remaining options will not result in an expression used in English. POINTS OF INTEREST: uncountable nouns, future tenses

6. CORRECT ANSWER:

b) had he

The blank requires the appropriate question tag. Since he’d is the contracted form of he had, a part of the expression had better, had is then also used in the question tag. Further, as had better is used here in the negative interrogative (he’d better not), as an advice form, the tag is affirmative. POINTS OF INTEREST: had better, question tags

page | 75

Mastering Proficiency Skills in English

7. He was __________ her roommate, a __________ named Shana. a) interested in / junior b) interested about / newcomer c) interesting in / rookie d) interesting about / novice

8. Eighteen Nazis were __________ after being convicted __________ crimes committed during World War II. a) hung / about b) hanged / of c) hung / of d) hanged / about

9. At first we didn’t notice the tall __________ who had come in __________ behind us, as he was dressed like the locals. a) foreigner / just b) intruder / rather c) stranger / quite d) invader / only

page | 76

Grammar/Vocabulary

7. CORRECT ANSWER:

a) interested in / junior

While any one of the four options offered to complete the second blank could be successfully used, with a resulting difference in meaning in each case, only one of the options offered for the first blank is a possible solution. The expression which needs to be completed is be interested in something or someone. POINTS OF INTEREST: adjectives ending in –ing/–ed, adjective + preposition

8. CORRECT ANSWER:

b) hanged / of

The verb hang is here used in the sense ‘be killed, usually as a punishment, by having a rope tied around the neck and the support taken away from under the feet’; when it is used with this meaning, the past participle form is typically hanged, as in (b) and (d), not hung as in (a) and (c). The second blank requires a preposition which collocates with the verb convict. Only of collocates with convict, as in (b) and (c), not a) and d) about. However, only (b) is a well-suited option for both blanks. POINTS OF INTEREST: regular/irregular verb hang, verb + preposition

9. CORRECT ANSWER:

a) foreigner / just

While any one of the three options offered for the first blank is possible, excluding d) invader which is commonly used in relation to military forces attacking a country, a) foreigner and c) stranger are most likely in the context of this sentence, the former meaning ‘someone coming from a country different from your own’ and the latter ‘someone you have never met before’. The second blank is followed by the preposition behind. To complete the blank it is necessary to choose one of the four adverbs. Some adverbs collocate with prepositions and some do not. In this context, the correct solution is a) just, an adverb which directs attention to the part of the sentence which immediately follows it, in this case it directs attention to the preposition behind. b) rather can be used in the sense ‘somewhat, to some degree’; c) quite can be used in the sense of ‘completely’ or ‘positively’, and even ‘rather’; finally, d) only has a number of meanings, e.g. ‘merely’, ‘solely’, ‘at the very least’, ‘in the final outcome’, ‘not before’, none of which are appropriate in this context.

page | 77

Mastering Proficiency Skills in English

10. Giving your child __________, appropriate and varied __________ teaches them to take responsibility. a) sensitive / awards b) senseless / prizes c) sensible / rewards d) insensitive / gifts

11. Not only __________, but he also set a new world record at __________ Olympics. a) did he won / the b) he won / c) did he win / the d) had he won / -

12. Jason __________ wearing an old coat of __________. a) turned in / Patricks b) turned up / Patrick’s c) turned around / Patrick d) turned away / Patricks’

page | 78

Grammar/Vocabulary

10. CORRECT ANSWER:

c) sensible / rewards

Of the four adjectives offered for the first blank, only the meaning of c) sensible ‘reasonable’ is appropriate for this sentence as a) sensitive means ‘easily upset/offended by other people’s behavior’ or ‘showing awareness and understanding of other people’s feelings’; its antonym is d) insensitive, meaning quite the opposite of the aforementioned, whereas b) senseless implies something ‘foolish, idiotic, pointless’. The intended meaning of the sentence, which could be paraphrased as ‘awarding your child sensibly, appropriately and variedly teaches them to take responsibility’, requires that the second blank be completed with c) rewards, ‘something valuable, such as money’ since a) awards include certificates, medals and prizes, which are often publicly presented. Furthermore, b) prizes imply winning a competition, or a game, while d) gifts are presents, generally speaking, which makes these two options inappropriate in this context. POINTS OF INTEREST: word formation by means of prefixes/suffixes

11. CORRECT ANSWER:

c) did he win / the

The first part of the structure not only … but also … has been placed at the beginning of the sentence; this requires inversion and, therefore, b) he won can be ruled out. Further, a) did he won must be ruled out too because it is ungrammatical - the form of the verb win used is the past participle, instead of the required bare infinitive with the auxiliary do. d) had he won is also not appropriate here because there is no indication that another action happened afterwards, for which the use of the Past Perfect Tense would be necessary; rather, the intended meaning is that this person both won and set the new world record. Furthermore, as the second blank precedes the noun Olympics, which requires the definite article, the choice between (c) and (d) has to be narrowed down to (c). POINTS OF INTEREST: inversion, definite article

12. CORRECT ANSWER:

b) turned up / Patrick’s

Only the meaning of the phrasal verb b) turn up, ‘appear’, fits the intended meaning of the sentence since a) turn in has many meanings, one of them being ‘go to bed’, c) turn around means ‘move something so that it faces opposite direction’, whereas d) turn away denotes ‘not allowing a person to enter a country, or some other place’. The second blank requires completion of the so-called ‘double genitive’ – of + GENITIVE/POSSESSIVE PRONOUN; here the genitive form of the proper noun Patrick is needed, that is Patrick’s. POINTS OF INTEREST: phrasal verbs, double genitive page | 79

Mastering Proficiency Skills in English

13. What does WYSIWYG __________? What You See Is What You Get, if I remember __________. a) stand for / correctly b) stand in / correct c) stand by / correctly d) stand up for / correct

14. __________ by lightning, the old oak started __________. a) Struck / to dry b) Having been stricken / to fade c) Stricken / to bloom d) Having been struck / to wither

15. The interviewer asked Jane __________ to tell him about her work __________ including all part-time jobs she __________. a) to precede / experiences / could have had b) to proceed / experience / might have had c) to precede / experience / would have had d) to proceed / experiences / should have had

page | 80

Grammar/Vocabulary

13. CORRECT ANSWER:

a) stand for / correctly

The meaning of the phrasal verb a) stand for ‘represent’ best fits in with the intended meaning of the sentence. Other phrasal verbs entail meanings that do not fit the context: b) stand in is commonly followed by prepositions for and with and has numerous meanings depending on the context. c) stand by, as an intransitive verb denotes ‘to be present/to remain apart or aloof’, while as a transitive verb it means ‘to remain loyal or faithful to’; finally, d) stand up for means ‘to defend against attack or criticism’. The second blank is preceded by the verb remember and therefore requires an adverb, correctly, as in (a) and (c), not an adjective (correct). POINTS OF INTEREST: phrasal verbs, adjectives/adverbs

14. CORRECT ANSWER:

d) Having been struck / to wither

The first blank requires an appropriate form of the verb strike. This verb has two past participles struck and stricken; the latter is only used in the sense ‘afflicted or overwhelmed by or as if by disease, misfortune, or sorrow’. It is, therefore, not appropriate here, which immediately rules out b) having been stricken and c) stricken. Further, as the action expressed by struck happened before the action expressed by the next verb, the PERFECT PARTICIPLE PASSIVE (d) and not the PAST PARTICIPLE (a) is required. The second blank requires an intransitive verb the meaning of which is appropriate in this context. The correct solution is d) to wither, meaning to ‘dry up and die’. The meaning of b) fade ‘to become paler, or hardly noticeable until it disappears’ and c) bloom ‘to produce flowers’ makes them unsuitable options, while a) dry cannot be used to refer to dying plants.

15. CORRECT ANSWER:

b) to proceed / experience / might have had

To correctly complete the first blank, b) and d) proceed ‘to go on in an orderly regulated way, to continue’ must be selected as the meaning of precede is ‘happen/come before something else’. The noun experience is to be used in this sentence in the sense ‘practical knowledge, skill, or practice derived from direct observation of or participation in events or in a particular activity’; when used in this sense, experience is used as an uncountable noun. This means that experiences, as in (a) and (d) can be ruled out. Finally, to complete the final blank an appropriate modal verb must be selected. Used with the PERFECT INFINITIVE, b) might, expresses speculation about past actions (it is not clear whether Jane actually did have any part-time jobs in the past). Another possibility is a) could have had, as COULD + PREFECT INFINITIVE can be used when it is not clear whether the action was performed or not. Neither c) would have had nor d) should have had are appropriate here; WOULD + PERFECT INFINITIVE is used to form the Perfect Conditional Tense, while SHOULD + PERFECT INFINITIVE is typically used to talk about an unfulfilled obligation or a sensible course of action which was neglected. POINTS OF INTEREST: modals in the past page | 81

Mastering Proficiency Skills in English

16. __________ an adult one must accept his __________ of responsibilities. a) Being / part b) To be / share c) Having been / bit d) To have been / piece

17. Hey, fancy __________ you here! Who __________ you’d come, knowing how much you hate these social __________. a) to see / would think / occasions b) seeing / would have thought / gatherings c) having seen / would think / events d) to have seen / would have thought / happenings

18. Mrs. Coward __________ her husband another cup when the air raid sirens __________. a) was spilling / went on b) was pouring / went off c) has been pouring / went on d) has been spilling / went off

page | 82

Grammar/Vocabulary

16. CORRECT ANSWER:

b) To be / share

Purpose is typically expressed by the INFINITIVE, as in this sentence, which can be paraphrased as ‘In order to be an adult one must accept…’ or ‘If one is to be an adult one must accept…’. The infinitive of purpose may either follow or precede the main verb, as is the case in b) To be. For this reason both a) being, the PRESENT PARTICIPLE, and c) having been, the PERFECT PARTICIPLE, are inappropriate in this context. As the PERFECT INFINITIVE in d) to have been, is used to talk about things that didn’t happen, it is also not suitable here. The second blank requires a noun which collocates with responsibilities and this is commonly share in (b) (note also shared responsibility/ies), not part in (a). Neither c) bit nor d) piece are appropriate here as responsibilities is a plural noun, while these two are usually used with uncountable nouns. POINTS OF INTEREST: infinitives

17. CORRECT ANSWER:

b) seeing / would have thought / gatherings

The first blank is preceded by fancy which is used here not as an adjective, but as a verb meaning ‘imagine’ and must therefore be followed by a GERUND. This rules out both a) to see and d) to have seen, as well as c) having seen, as this is the PERFECT GERUND, used to denote completion of an action in the past, which is not the case in this sentence (note the use of hey implying the action is taking place in the present). The second blank is followed by you’d come, where you’d is the contracted form of you would, not you had. To introduce guesses or to comment on a past action should/would have thought is often used, but not would think, as in (a) and (c). Finally, the third blank requires the choice of an appropriate noun. In fact, any one of the four options offered is possible here, d) happenings being slightly less probable than the other three. POINTS OF INTEREST: gerund

18. CORRECT ANSWER:

b) was pouring / went off

The correct verbal form for the first blank is pour. Further, as the action being described began before the point in time identified in the subordinate clause (when the air raid sirens…) and possibly continued after it, the Past Continuous Tense (b) is required, not the Present Perfect Continuous (c). The second blank is to be completed by choosing the appropriate phrasal verb, which, in this case, is go off ‘[of a sound-creating device] to make its noise’. POINTS OF INTEREST: continuous aspect, phrasal verbs

page | 83

Mastering Proficiency Skills in English

19. What has to be borne in mind __________ cosmetic surgery is that there are no guarantees one’s expectations will be __________ once the bandages __________. a) in regard with / met / have been taken off b) as regards / fulfilled / are taken off c) with regard to / filled / will be taken off d) regarding / matched up to / are going to be taken off

20. The police __________ still looking for the little girl who was __________ in Spain last summer. a) is / abandoned b) are / abducted c) is / adopted d) are / adapted

21. I clicked the button, the computer screen _______ blank and all I ________ was pray for the information to be _________ restored. a) gone / can do / miraculous b) went / could do / miraculously c) had gone / could do / miraculously d) has gone / can do / miraculous

page | 84

Grammar/Vocabulary

19. CORRECT ANSWER:

b) as regards / fulfilled / are taken off

The first blank requires an appropriate focusing expression with regard. Of the four options offered, three are possible: b) as regards, c) with regard to and d) regarding. The expression in a) in regard with is not a possible alternative, as the correct expression is in regard to. The second blank requires a verb which collocates with the noun expectations as subject; either a) met or b) fulfilled is possible but not c) filled or d) matched up to. The second blank is followed by once, which is being used here as a conjunction meaning ‘as soon as’. This means that the third blank is part of a time clause beginning with once. In this type of clause, future forms (the Future Simple, Present Continuous, be going to) are not used and thus it is possible to rule out c) will be taken off and d) are going to be taken off. Either the Present Perfect, as in a) have been taken off, or the Simple Present, as in b) are taken off, is possible, yet only (b) is a well-suited solution to all three blanks. POINTS OF INTEREST: collocations, tenses

20. CORRECT ANSWER:

b) are / abducted

The noun police is one of several nouns in English which are plural but do not end in –s. It is therefore possible to rule out is, as in (a) and (c), as a potential correct answer to the first blank. The second blank might be completed by inserting either a) abandoned or b) abducted; c) adopted might be possible, but is not very likely in this context, while d) adapted is not possible as this verb denotes ‘changing something to make it suitable for a new purpose or situation’. Therefore, only (b) meaningfully completes the sentence. POINTS OF INTEREST: plural nouns

21. CORRECT ANSWER:

b) went / could do / miraculously

The sentence clearly illustrates the use of the Simple Past Tense for describing a series of successive actions. Accordingly, the first and second blank require the verb forms in the Simple Past Tense and the only correct option is (b). The third blank should be filled in with an adverb that modifies the passive infinitive to be restored which is the case in (b) and (c). The adjective form a) and d) miraculous cannot modify this verb, as adjectives commonly appear before nouns or after link verbs. Therefore the only answer that completes all three blanks correctly is (b). POINTS OF INTEREST: Simple Past, adjectives/adverbs

page | 85

Mastering Proficiency Skills in English

22. If you had tried __________ to get into the team, I’m sure you __________. a) hard enough / will have succeeded b) hardly enough / would succeed c) hardly enough / will succeed d) hard enough / would have succeeded

23. Except for the Sun, all stars are too far from the Earth for their distances __________ in miles or kilometers. a) to conveniently measure b) to be conveniently measured c) to measure conveniently d) to be conveniently measure

24. I’d rather you __________ the child that Santa Claus doesn’t really exist. a) tell b) had told c) told d) would tell

page | 86

Grammar/Vocabulary

22. CORRECT ANSWER:

d) hard enough / would have succeeded

The first blank can only be completed with hard enough, as in (a) and (d), the adverb hard, meaning ‘with great or utmost effort or energy’, modified by the adverb of degree enough as hard in one of the words which can function as both an adjective and an adverb (similar to fast and late). The meaning of hardly enough ‘a barely sufficient number, quantity, or amount’, as in (b) and (c), is not appropriate here. The correct solution to the second blank is d) would have succeeded as the sentence begins with If you had tried…, the subordinate clause of the third conditional. POINTS OF INTEREST: hard/hardly, conditional sentences

23. CORRECT ANSWER:

b) to be conveniently measured

To complete this sentence, attention must be paid to the fact that the noun phrase their distances is preceded by for. This indicates that it is necessary to complete the structure for + OBJECT + INFINITIVE. It is important to remember that the PASSIVE INFINITIVE, as in b) to be conveniently measured, is needed in this structure as it can be paraphrased as ‘so that we could measure their distances’; to-INFINITIVE in (a) and (c), therefore, has to be ruled out as the verb to measure is a transitive verb and no object is provided after it, as well as d) to be conveniently measure which is ungrammatical employing bare infinitive, measure, instead of past participle. POINTS OF INTEREST: passive infinitive

24. CORRECT ANSWER:

c) told

It is possible to use rather to talk about preference as part of the expression would rather, meaning ‘would prefer to’ (e.g. I’d rather stay at home than go out); would rather is often followed by the BARE INFINITIVE, i.e. the infinitive without to. However, would rather can be used to say that a person would prefer somebody to do something, that is, when there are two subjects in a sentence: SUBJECT + WOULD RATHER + SUBJECT, the structure which is then followed by the Past Simple Tense. This is commonly referred to as SUBJUNCTIVE. POINTS OF INTEREST: would rather, subjunctive

page | 87

Mastering Proficiency Skills in English

25. The __________ had moved down from the hills after the government __________ the river, to create a __________ waterfront for tourists. a) local people / closed / picnicking b) settlers / bordered / picnicing c) villagers /dammed / picnicking d) citizens / stopped / picnicing

26. Juliette saw a sign saying TOILETS, hurried in and asked someone: ‘Is this cubicle __________?’ a) taken b) occupied c) busy d) free

27. Twelve __________ people gathered to greet the __________ president. a) thousands / elective b) thousands / electing c) thousand / elected d) thousand / elections

page | 88

Grammar/Vocabulary

25. CORRECT ANSWER:

c) villagers / dammed / picnicking

The first blank could be completed by any of the four nouns offered, b) settlers and d) citizens being the least likely correct answers since the former refers to ‘people who have gone to live in a new country’, and the latter to ‘a group of people who live in towns or cities’. The answer which best completes the second blank based on the sentence context is c) dammed, meaning ‘built a dam across the river’. Other options do not make sense here. To complete the third blank, the correct spelling of the PRESENT PARTICIPLE of the verb picnic, that is a) and c) picnicking, is needed.

26. CORRECT ANSWER:

b) occupied

The meaning of the noun cubicle is ‘a small partitioned space’. Juliette’s question could be paraphrased as ‘Is this cubicle being used by somebody?’ The adjectives c) busy and d) free are not appropriate here. Neither is the adjective a) taken, which may be used to describe a seat or chair, for example, which has been obtained or secured for use. As one meaning of b) occupied is ‘being used by somebody’, it is the best solution to this blank.

27. CORRECT ANSWER:

c) thousand / elected

As the words dozen, hundred, thousand, million are never made plural when used to refer to a definite number (e.g. two dozen eggs, four hundred dollars, two hundred thousand inhabitants), it is possible to immediately rule out thousands, as in (a) and (b), as an appropriate solution to the first blank because of the numeral twelve which precedes it. The second blank must be completed by a form which can modify the noun president. Either PRESENT or PAST PARTICIPLES can be used as adjectives, i.e. can modify nouns, but with a difference in meaning. PRESENT PARTICIPLE adjectives have the meaning ‘having this effect’, hence the present participle electing in (b) is not appropriate here. PAST PARTICIPLES adjectives have the meaning ‘affected in this way’; the past participle elected in (c) can, therefore, be used to complete the second blank. As one of the meanings of elective in (a) is ‘chosen or filled by popular election’, this adjective is also a possibility. The noun elections in (d) can be ruled out as it is in the plural (nouns which appear in front of other nouns and modify their meaning do not typically have a plural form, e.g. class president, behavior problems). Only answer (c), however, meaningfully completes both blanks.

page | 89

Mastering Proficiency Skills in English

28. A commuter is a person who has to travel to __________ on a daily __________. a) job / basis b) work / bases c) job / bases d) work / basis

29. Don’t be nosy! Don’t you know that curiosity killed the __________? a) dog b) cat c) mouse d) rat

30. The team was exhausted after the all-day practice. They decided to __________. a) call it a day b) call it off c) pay a call d) call in

page | 90

Grammar/Vocabulary

28. CORRECT ANSWER:

d) work / basis

In addition to the difference in meaning between the nouns job and work, and the fact that work is an uncountable noun, whereas job is a countable noun, there is another important difference. Unlike job, the noun work can be used without an article or other determiner following the preposition to, as in this sentence, and the prepositions at and from. The noun in a) and d) basis can be used to mean ‘the way something is done or organized’ and is typically used in the singular in this sense, e.g. on a weekly basis, on a permanent basis, on a voluntary basis, on a commercial basis. Its plural form is b) and c) bases.

29. CORRECT ANSWER:

b) cat

The correct expression is curiosity killed the cat, used in order to tell someone ‘that they should not try to find out about something which does not concern them’.

30. CORRECT ANSWER:

a) call it a day

The meaning of a) call it a day is ‘to quit work and go home; to say that a day’s work has been completed’. The meaning of the phrasal verb b) call off is ‘abandon something already in progress; cancel something not yet started’; this phrasal verb is not appropriate here because the first sentence clearly states that the all-day practice was over when the team made their decision (after the all-day practice). To c) pay a call to someone means ‘visit’. The phrasal verb d) call in is transitive (i.e. requires a direct object – call someone in) and no object has been provided; one of its meanings is ‘send for/ask someone to come to the house to perform some service’.

page | 91

Mastering Proficiency Skills in English

TEST 5 1. It is the __________ reports that have been misplaced. Find them as soon as possible! a) Prince of Wales b) Prince of Wales’s c) Prince’s of Wales d) Prince of Wales’

2. Occasionally I try __________ them by phone, but they are __________ ever at home. a) to call / almost b) to get / rarely c) to reach / hardly d) to contact / seldom

3. Situated in the Black Hills of __________ South Dakota, the __________ stands out from the mountainside, which __________ high above the surrounding farmland. a) - / memorandum / raises b) the / statue / raises c) a / monument / rises d) - / memorial / rises

page | 92

Grammar/Vocabulary

1. CORRECT ANSWER:

b) Prince of Wales’s

When the possessive of names consisting of several words is formed, the last word takes ‘s irrespective of its final sound. POINTS OF INTEREST: double genitive

2. CORRECT ANSWER:

c) to reach / hardly

While a) to call someone by phone, c) to reach someone by phone and d) to contact someone by phone are all possible, b) to get someone by phone is not; to get someone on the phone should be used instead. The second blank requires an adverb which can modify the adverb ever. As the intended meaning of the second clause (but they are…) can be paraphrased as ‘but they are almost never at home’, that is, the second clause is negative, the adverb c) hardly must be selected; it is the only adverb offered which can make a clause or sentence negative and can modify the adverb ever. The adverbs b) rarely and d) seldom can also make a clause or sentence negative, but they are not adverbs of degree, unlike c) hardly, and cannot, therefore, modify other adverbs. Finally, a) almost is an adverb of degree but it cannot make a sentence negative.

3. CORRECT ANSWER:

d) - / memorial / rises

The first blank must be completed with the zero article (-) as South Dakota is a state in the United States of America and no article is typically used with the names of counties, states and countries. An appropriate noun must be selected to complete the second blank. Of the four solutions offered, only one is not appropriate; the meaning of a) memorandum is ‘informal record, written reminder’. Finally, to complete the third blank it is necessary to choose between the verbs raise and rise. The meaning of raise is ‘to cause or help to rise to a standing position’ and it is a transitive verb (it requires a direct object); it is, therefore, not appropriate here (note that the verb is not in the passive nor is there a direct object following the verb). The verb rise, the correct solution to the third blank, is intransitive and can be used to mean ‘to extend above other objects’. Only (d), therefore, appropriately completes all three blanks. POINTS OF INTEREST: articles, regular verb raise/irregular verb rise

page | 93

Mastering Proficiency Skills in English

4. __________ of the contestants was able to __________ the Olympic record. a) No one / beat b) None / break c) Neither / beat d) Not one / break

5. American subjects, speaking of foreign lands, often __________: ‘If only their political systems _______’. a) state / like ours were b) argue / were like our c) exclaim / were like ours d) say / like our were

6. John __________ as sick as he __________. a) isn’t probably / was used to being b) isn’t probable / was used to be c) probably isn’t / used to be d) probable isn’t / was used to

page | 94

Grammar/Vocabulary

4. CORRECT ANSWER:

d) Not one / break

To complete the first blank correctly, attention must be paid to the fact that the first blank is part of the subject of this sentence and the verb (was able…) is in the singular. Both c) neither and d) not one can be used with a verb in the singular and followed by of + an article/ possessive adjective/demonstrative + a noun in the plural. On the other hand, b) none requires a verb in the plural, while a) no one cannot be followed by of + NOUN. To complete the second blank it is necessary to choose between the verbs a) and c) beat and b) and d) break. The correct collocation is break a record so only (d) is a well-suited solution to both blanks. POINTS OF INTEREST: no one/none, collocations

5. CORRECT ANSWER:

c) exclaim / were like ours

All options offered for the first blank are appropriate here. The second blank does not require an inversion, as is the case in (a) and (d), since it is a part of If only + subject + verb + nominal predicate, which is the case in (b) and (c). Furthermore, the choice between the possessive pronoun ours (c), and the possessive adjective our (b), is governed by the fact that no noun is provided after the blank, so the use of an adjective is ungrammatical. POINTS OF INTEREST: if only + subjunctive, possessive pronouns/adjectives

6. CORRECT ANSWER:

c) probably isn’t / used to be

The first blank should be completed with a verb and an adverb, not an adjective as adjectives appear before nouns, not verbs. It is, therefore, possible to immediately rule out b) isn’t probable because probable is an adjective, as well as d) probable isn’t. Finally, the adverb probably goes in mid-position, i.e. between the verb and its subject, and never in end-position, i.e. after the verb it modifies. This rules out a) isn’t probably, leaving (c) as the only correct answer. The second blank requires the correct use of the structure used to + infinitive. This structure refers only to past habits and states, as is the case in this context. There is a difference in meaning between used to + infinitive (c) and be + used to + -ing/noun phrase (d) as the latter refers to an experience that one has been in so frequently that it is no longer strange to him/her. This excludes (d) as the potential correct answer. Other options are ungrammatical, so only (c) fits this blank. POINTS OF INTEREST: word order, adjectives/adverbs, used to + inf./-ing page | 95

Mastering Proficiency Skills in English

7. Andy Warhol became famous __________ painting __________ tin of soup. a) of / the b) for / a c) in / a d) at / the

8. __________ to avoid the traffic jam downtown we went __________ a roundabout route. a) So as / via b) In order / across c) So / over d) So that / down

9. It is interesting to note that the color of her eyes varies __________ the color of her __________. a) depending on / clothing b) depending of / cloths c) in dependence of / cloth d) with dependence of / clothes

page | 96

Grammar/Vocabulary

7. CORRECT ANSWER:

b) for / a

The adjective famous can be followed by the preposition as or the preposition for. The meaning of famous as is ‘famous in the capacity, character, condition, or role of’ (e.g. He was famous as both an author and a critic), while the meaning of famous for is ‘famous as a result of’. As Andy Warhol became famous as a result of painting certain objects, as well as people, b) for is appropriate here. The article needed to complete the second blank is the indefinite article because it is not any particular tin of soup that is being referred to. Even if the sentence had been more specific and had said ‘painting ____ tin of Campbell’s soup’, the indefinite article would have been needed as, again, it is not any particular tin of Campbell’s soup that is being referred to, but any tin of soup selling under this brand. POINTS OF INTEREST: adjective + preposition, indefinite article

8. CORRECT ANSWER:

a) So as / via

The first blank is followed by the infinitive preceded by to (to avoid) and can, therefore, be completed by either a) so as (so as to avoid) or b) in order (in order to avoid). The other two options cannot be used in this way. The second blank requires an appropriate preposition. As the intended meaning of the prepositional phrase to be completed is ‘by way of an indirect line of travel’, only the preposition via in (a), meaning ‘by way of’, is a suitable solution. POINTS OF INTEREST: clauses of purpose, prepositions

9. CORRECT ANSWER:

a) depending on / clothing

The verb depend is typically followed by the preposition on as in (a), so it is possible to rule out b) depending of. Further, the noun dependence cannot be used with any combination of preceding and/or following prepositions to mean ‘to be determined, based, or contingent’ as depend followed by on can be. It is therefore possible to also rule out c) in dependence of and d) with dependence of as possible solutions to the first blank. The second blank can be completed by either a) clothing or d) clothes, meaning ‘garments’, but not by b) cloths or its singular form c) cloth, meaning ‘material, fabric’, as neither of these has the aforementioned meaning. POINTS OF INTEREST: verb + prepositions, confusable words

page | 97

Mastering Proficiency Skills in English

10. Seldom __________ around here at the premieres of new movies. Consequently, only __________ money is made and the cinema is __________ being closed down. a)

celebrities have been seen / a little / in danger of

b) celebrities are seen / little / on the brink of c) have celebrities been seen / a little / on the verge of d) have celebrities seen / little / under threat of

11. While seeking possible solutions to the problem, __________ Rick __________ a great idea. a) out of nowhere / made up b) out of space / made up c) out of the dark / came up with d) out of the blue / came up with

12. You will never __________ with what you have done now! My watch is completely ruined and I intend to complain to your parents. a) get on b) get by c) get off d) get away

page | 98

Grammar/Vocabulary

10. CORRECT ANSWER: of

c) have celebrities been seen / a little / on the verge

The first blank is preceded by seldom, placed at the beginning of the sentence for emphasis; when seldom is used in this position, inversion of the subject and verb is required. This means that a) celebrities have been seen and b) celebrities are seen can immediately be ruled out. Further, the first blank requires a verb in the passive, as celebrities are the ones who are seen by others. Therefore, d) have celebrities seen can also be ruled out. The second blank is preceded by only. Placed in front of a little or a few the adverb only serves to emphasize that the number (a few) or amount (a little) is small in the speaker’s/writer’s opinion. It is therefore possible to rule out little in (b) and (d). Finally, the third blank can be completed by any one of the four options offered, yet only (c) meaningfully completes all three blanks. POINTS OF INTEREST: inversion, little/a little

11. CORRECT ANSWER:

d) out of the blue / came up with

The meaning of d) out of the blue is ‘suddenly, without warning’; a) out of nowhere also has this meaning, but is typically used with the verbs appear, come and materialize. Neither b) out of space nor c) out of the dark are idiomatic expressions in English and their meaning does not fit in with the intended meaning of the sentence. To complete the second blank it is necessary to choose between a) and b) make up and c) and d) come up with. None of the numerous meanings of the phrasal verb make up fit the context of this sentence. The phrasal verb come up with, meaning ‘manage to find or improvise something’, is therefore appropriate.

12. CORRECT ANSWER:

d) get away

Note that the blank is followed by the preposition with. The meaning of the phrasal verb get away with is ‘do something without being punished for it, or suffering any other bad consequences’ which is suitable here. a) get on has many meanings including ‘to have a friendly relationship with someone or to continue doing something’; b) get by means ‘to survive, to cope with a difficult situation’, whereas c) get off denotes that ‘a law/rule was broken but that the person was not punished, or got a very small punishment’.

page | 99

Mastering Proficiency Skills in English

13. __________ by a throng of spectators, the runners __________ the finishing line. a) Surrounding / neared b) Surrounded / approached c) Having surrounded / reached d) Having been surrounded / came close

14. It __________ Jane whom you saw in the street yesterday. She went to __________ Seychelles, remember? a) can’t be / the b) can’t have been / the c) couldn’t be / d) couldn’t have been / -

15. __________ in advance what I had to buy, I __________ much time at the shopping mall. a) To know / didn’t need to spend b) Having known / needn’t have spent c) Knowing / didn’t need to spend d) To have known / needn’t have spent

page | 100

Grammar/Vocabulary

13. CORRECT ANSWER:

b) Surrounded / approached

The first blank is part of a participle clause whose subject is mentioned after the comma (the runners) because it is the same in both clauses. Its meaning could be paraphrased as ‘The runners were surrounded by a throng of spectators’ or ‘The runners had been surrounded’ (SUBJECT + PASSIVE VERB). For this reason the PAST PARTICIPLE, as in b) surrounded, or the PERFECT PARTICIPLE PASSIVE, as in d) having been surrounded, can be used. As neither the present participle nor the perfect participle active can be used to replace a SUBJECT + PASSIVE VERB, both a) surrounding and c) having surrounded can be ruled out. The second blank can be completed by three of the four options offered. Only d) came close is not a possible solution as the adverb close must be followed by the preposition to here; came close to the finishing line means ‘approached very near to the finishing line’. The correct answer must, then, be (b) as it is the only one which makes it possible to complete both blanks correctly. POINTS OF INTEREST: participles

14. CORRECT ANSWER:

b) can’t have been / the

Note the use of yesterday in the first sentence; yesterday clearly indicates that the time reference is past and that a modal verb with a PERFECT INFINITIVE (e.g. have been) must be used. It is possible to use all modal verbs except shall together with perfect infinitives to talk about the past. Since neither makes use of a perfect infinitive, both a) can’t be and c) couldn’t be can be ruled out. The second blank must be completed with the definite article as it is required in front of the names of island groups, such as the Seychelles, which makes (b) the correct option for both blanks. POINTS OF INTEREST: modals in the past, definite article

15. CORRECT ANSWER:

c) Knowing / didn’t need to spend

The present participle can be used to replace as/since/because + SUBJECT + VERB. Used this way it helps explain the action which follows. Neither the infinitive, a) to know nor the perfect infinitive, d) to have known can be used this way. As the first clause in this sentence can be paraphrased as ‘Since I knew in advance what I had to buy’, either b) having known or c) knowing could be the correct solution to the first blank. The difference between needn’t + PERFECT INFINITIVE (e.g. needn’t have spent) and need as an ordinary lexical verb (e.g. didn’t need to spend) is that the former is used to express an unnecessary action which was nevertheless performed, while the latter is used to express an action which was not necessary and its use does not necessarily imply that the action was performed. If the first blank is completed correctly, it becomes obvious that only didn’t need to spend makes sense in the context of this sentence. POINTS OF INTEREST: participles, infinitives, the verb need page | 101

Mastering Proficiency Skills in English

16. She was sorry __________ Tim when he was in Paris and hoped he wasn’t mad __________ her. a) not to phone / about b) not to have phoned / with c) not to phone / on d) not to have phoned / at

17. Once a ceasefire was announced, it __________ long before our troops __________ in France. a) wasn’t / landed b) hadn’t been / departed c) isn’t / settled d) won’t be / arrived

18. A hunger strike __________ organized unless the local government __________ wages. a) is / raise b) will be / rises c) is to be / raises d) is going to be / rise

page | 102

Grammar/Vocabulary

16. CORRECT ANSWER:

d) not to have phoned / at

The PERFECT INFINITIVE, e.g. to have phoned, can be used to express an earlier action, that is, an action which happened before the action expressed by the main verb. In other words, She was sorry … when he was in Paris makes it clear that ‘she’, the subject, didn’t phone Tom during the time he was in Paris and was later sorry about this. Therefore, not to phone can be ruled out and the perfect infinitive is the correct solution to the first blank. The second blank requires an appropriate preposition. The adjective mad ‘furious’ can be followed by about or at, but with a difference in meaning. The meaning of mad at is ‘angry with’, the meaning of mad about is ‘fond of’. Clearly d) at is the correct answer for the second blank. POINTS OF INTEREST: perfect infinitive, adjectives + prepositions

17. CORRECT ANSWER:

a) wasn’t / landed

Past time reference is established in this sentence by the Simple Past Tense in once a ceasefire was announced. The action expressed by the main verb is a later action in relation to the action expressed by the subordinate clause. The first blank, therefore, must be completed using the Simple Past Tense, i.e. a) wasn’t. To complete the second blank, an appropriate verb must be selected. Of the four options offered, only b) departed is not an appropriate choice because the verb depart is typically followed by for or from and never by in when the preposition is followed by a placename. POINTS OF INTEREST: past tenses

18. CORRECT ANSWER:

c) is to be / raises

Note that the sentence makes use of unless, meaning if not, which implies that ‘A will happen if it is not stopped by B’. It is therefore not possible to complete the first blank with a) is. Any one of the three remaining options is a possible solution to the first blank. Further, note that be to + INFINITIVE, as in c) is to be, is frequently used for official/formal arrangements and in the newspaper, radio or television reports to refer to future events. It expresses near certainty that what is forecast will happen. The second blanks is positioned between two noun phrases, the first of which, the local government, is the subject of the verb, while the second, wages, is the object. It therefore requires the verb raise rather than rise because raise is a transitive verb, whereas rise is intransitive. POINTS OF INTEREST: be to + infinitive, regular verb raise/irregular verb rise page | 103

Mastering Proficiency Skills in English

19. Many __________ innovations, such as the telephone, appear to be the result of sudden bursts of inspiration, when in fact they ______ by years of hard work. a) valuable / were preceded b) valueless / had been preceded c) invaluable / had been preceded d) valued / were preceded

20. One of the guests on the talk show talked about how a patchwork quilt that looked quite ordinary __________ on a bed __________ into a work of abstract art if hung on a wall. a) laying / would be transformed b) lying / may be transformed c) lying / might be transformed d) laying / will be transformed

21. The police __________ for the suspect for more than two months now but he is nowhere __________. a) has been looking / being found b) has looked / being found c) have been looking / to be found d) had looked / to be found

page | 104

Grammar/Vocabulary

19. CORRECT ANSWER:

c) invaluable / had been preceded

The first blank needs to be completed by a form which has value as its base and fits the context. Because it is obvious that innovations have been preceded by hard work, this requires an adjective that expresses the meaning ‘having great value’. Although the prefix in- generally has a negative meaning ‘not’, (compare active-inactive, comprehensible-incomprehensible), c) invaluable meaning ‘priceless’ or ‘extremely valuable, precious’ is the most appropriate option. d) valued, a past participle of the verb to value which means ‘to estimate or assign the monetary worth of something’, is also possible here, as well as a) valuable ‘having monetary value (having a good price)’. Only answer b) valueless ‘something of no or little value’ does not fit the context. To complete the second blank correctly, it is necessary to note that one action preceded another (innovations were the result of years of hard work). Therefore, the Past Perfect Tense is needed, as in (b) and (c), not the Simple Past Tense, as in (a) and (d), to complete the sentence grammatically. Only answer (c) fits the context of both blanks.

20. CORRECT ANSWER:

c) lying / might be transformed

To complete the first blank, the choice must be made between the present participle of the verb lie (lying) and the verb lay (laying). The meaning of lie is ‘to be or to stay at rest in a horizontal position’ and it is an intransitive verb. On the other hand, lay is a transitive verb, meaning ‘to put or set something down’. Since no object is provided after the first blank, the present participle of lie, as in (b) and (c), is the only appropriate option here. Filling in the second blank completes a conditional sentence in reported speech (note one of the guests…talked about…). The conditional sentence in question is a type 1 conditional (‘a patchwork quilt…will/may be transformed into a work of abstract art if (it is) hung on a wall’), whose tenses change in reported speech due to the rule of the Sequence of Tenses. For the above reasons, although (a) and (c) can be correct answers, only c) might be transformed can be used to complete both blanks. POINTS OF INTEREST: irregular verbs lie/lay, Sequence of Tenses

21. CORRECT ANSWER:

c) have been looking / to be found

The subject of the verb needed to complete the first blank is police, a noun which, meaning police officers, takes a plural verb. Further, note the use of for + ‘a period of time’ + now in this sentence (for more than two months now); it clearly indicates that the Present Perfect Tense is required. It is, for the reasons just mentioned, possible to rule out all options offered for the first blank apart from c) have been looking. The PASSIVE INFINITIVE of the verbs see and find are commonly used after be followed by nowhere, e.g. he is nowhere to be found, while a) and b) being found is ungrammatical in this context (the verb find is not normally used in the continuous form). POINTS OF INTEREST: plural nouns, Present Perfect Continuous, passive infinitive page | 105

Mastering Proficiency Skills in English

22. I wish you __________ making __________ comments about my parents before you go too far. a) had stopped / unrespectful b) stopped / unrespectfull c) could have stopped / disrespectfull d) would stop / disrespectful

23. There comes __________ time in life when the past is __________ and you must __________ your bridges. a) - / too many of a burden / cross b) a / too many a burden / pass c) - / too much a burden / destroy d) a / too much of a burden / burn

24. He blindly follows every step she takes. We can conclude that generally he __________. a) bites the hand that feeds him b) gives her a helping hand c) eats out of her hand d) raises his hand against her

page | 106

Grammar/Vocabulary

22. CORRECT ANSWER:

d) would stop / disrespectful

Note that before you go too far establishes present time reference for this sentence. Therefore, the verb following wish must be in the Simple Past Tense or would + INFINITIVE could also be used, and is, in this case. This rules out both a) had stopped and c) could have stopped. The second blank requires an appropriate adjective. Of the four options offered, only d) disrespectful is an actual word in English. The remaining three are Englishlike nonwords. The noun meaning ‘lack of respect’ is disrespect and an adjective can be formed by adding the ending –ful (always spelled with a single letter ‘l’) to this noun. POINTS OF INTEREST: subjunctive, word formation by means of prefixes/suffixes

23. CORRECT ANSWER:

d) a / too much of a burden / burn

The first blank requires the indefinite article because the noun phrase time in life could be any one of a number of periods in a person’s lifetime. The noun burden which is to be used to complete the second blank is a countable noun. Too much is used before uncountable nouns in the sense ‘more than enough (of)’, but it can also appear before countable nouns in the singular when too much is followed by the preposition of and the indefinite article (too much of a). On the other hand, too many can only be used in front of plural nouns (which means these nouns cannot be preceded by the indefinite article) and cannot be followed by the preposition of. Therefore only d) too much of a burden can be used to complete the second blank. The correct expression required for the third blank is burn one’s bridges ‘make decisions which cannot be changed in the future’. POINTS OF INTEREST: indefinite article, idioms

24. CORRECT ANSWER:

c) eats out of her hand

This expression (c) means ‘to do exactly what someone says, to grovel to someone’. a) bite the hand that feeds one denotes ‘to do harm to someone who does good things for you’; b) give someone a helping hand means ‘to provide someone with (physical) help’, while c) raise a hand against someone refers to ‘to threaten (to strike) someone’.

page | 107

Mastering Proficiency Skills in English

25. __________ I tried my best to complete the task, even though I failed. a) Break the news! b) Give me a break! c) Break the ice! d) Break your word!

26. Is your handsome next-door neighbor __________? ~ As a matter of fact, he’s a __________. a) divorced / bachelorette b) engaged / fiancée c) single / widower d) available / spinster

27. __________ me a hand with these arithmetical calculations, will you? You’re an expert. a) Borrow b) Present c) Provide d) Lend

page | 108

Grammar/Vocabulary

25. CORRECT ANSWER:

b) Give me a break!

Option b) Give me a break! meaning ‘relief from annoyance’ is used to express exasperation or irritation. Other idioms do not fit the context: a) Break the news means ‘to make some information publicly known’; c) Break the ice means ‘to do or say something that makes people feel less shy or nervous in a social situation’, while d) Break your word is an Englishlike nonexpression.

26. CORRECT ANSWER:

c) single / widower

Of the four options offered for the first blank, three can be used to describe a man’s marital status, d) available being the least likely solution because it usually implies ‘not being busy and therefore free to talk or do a particular task’. As for the second blank, only one option can be used when referring to a man: c) widower. Note that b) fiancée is a noun used only to refer to an engaged woman, whereas fiancé can be used to refer to an engaged man. Likewise, a) bachelorette can be used to refer to an unmarried woman, whereas bachelor refers to an unmarried man. Finally, the meaning of d) spinster is ‘a woman who seems unlikely to marry’.

27. CORRECT ANSWER:

d) Lend

The expression lend someone a hand means ‘to give someone some help, not necessarily with the hands’. None of the other verbs collocates adequately with the phrase … someone a hand. It is important to note the difference between the verbs lend and borrow as they are easily confusable words: b) borrow means ‘to receive with the implied or expressed intention of returning the item in question’, whereas d) lend denotes ‘to give for temporary use on condition that the same or its equivalent be returned’. POINTS OF INTEREST: verbs lend/borrow

page | 109

Mastering Proficiency Skills in English

28. The Prime Minister __________ that the political party’s slogan at the convention in the Hague was: ‘__________ of the world unite in the fight for the common good’. a) said me / person b) told me / peoples c) spoke me / persons d) talked to me / people

29. The news of the earthquake in Indonesia __________ not encouraging. Buildings have been __________ damaged and rescue teams are still searching for __________. a) is / terribly / injured b) are / extremely / evacuees c) is / severely / survivors d) are / seriously / refugees

30. Ever since he was hired, Jonathan has been one of the __________ and highest-paid __________ in our company. a) diligentest / employees b) diligentest / employers c) most diligent / employers d) most diligent / employees

page | 110

Grammar/Vocabulary

28. CORRECT ANSWER:

b) told me / peoples

To complete the first blank a reporting verb must be selected. Note that the first blank is followed by a that-CLAUSE (that the political party’s…was…). Of the four verbs offered, only the reporting verbs say and tell can be followed by a that-CLAUSE, so c) spoke me and d) talked to me can be ruled out. The verb tell can be followed directly by a pronoun which is the verb’s indirect object, as in b) told me. However, a pronoun which is the indirect object of the verb say must be preceded by the preposition to (e.g. said to me); it is therefore also possible to rule out a) said me. The second blank could be completed by either d) people or c) persons, but also by b) peoples, a plural noun meaning ‘a body of persons that are united by a common culture, tradition, or sense of kinship, that typically have common language, institutions, and beliefs, and that often constitute a politically organized group’ but not by a) person because the noun is in the singular. Only (b), however, is a well-suited solution to both blanks.

29. CORRECT ANSWER:

c) is / severely / survivors

The noun news always takes a verb in the singular, which immediately rules out are in (b) and (d). Any one of the four options offered can be used to complete the second blank (note also that the adjectives terrible, extreme, severe and serious can all modify the noun damage). Finally, the third blank must be completed with survivors, as in (c), because a) injured would have to be preceded by the definite article to denote all the people who were injured, whereas b) evacuees and d) refugees do not make sense in this context due to their meaning, the former denoting ‘people sent away from a dangerous place to somewhere safe’ and the latter ‘people who have been forced to leave their homes or their country’. POINTS OF INTEREST: uncountable nouns

30. CORRECT ANSWER:

d) most diligent / employees

To complete the first blank it is necessary to choose the correct superlative form of the adjective diligent. Like all adjectives of three or more syllables, it forms the superlative using most. Therefore, only c) and d) most diligent can be used to complete the first blank correctly. The second blank is to be completed by choosing either employers or employees. Nouns ending in –er can have the meaning ‘one that produces or yields’ (hence an employer is someone who produces employment), while nouns ending in –ee can have the meaning ‘recipient or beneficiary of (a specified action)’ (hence an employee is someone who is employed). The context (ever since he was hired…) makes it clear that only employee can be used to complete the second blank appropriately. With the correct answer for the first blank, only (d) fits both blanks. POINTS OF INTEREST: superlative of adjectives, nouns ending in –er/–ee

page | 111

READING COMPREHENSION In this section, by choosing A, B, C, or D students will either complete statements which retell information given in the relevant passage, determine what a pronoun in the text refers to, or provide synonyms or antonyms for expressions used in the reading passage. Students are advised to carefully read each passage several times, paying special attention to how the argument, narrative or description in the passage unfolds. One final reading is suggested after students have thoroughly considered and understood the questions at the end of the passage.

Mastering Proficiency Skills in English

TEST 1 The birth of twins is a phenomenon of nature that causes immediate interest. Throughout history, however, this interest was not always positive. In the past, some cultures considered the birth of twins as unnatural or evil, and the mother and her babies were put to death. On the other hand, other cultures welcomed twins because twins were believed to possess supernatural magical powers. Today modern societies do not consider twins as either unnatural or supernatural, but twins do continue to fascinate people. Scientists, too, have developed a keen interest in twins. Through the study of twins, scientists hope to learn something about the roles of heredity and environment in shaping human lives. Children inherit characteristics from their parents through genes, microscopic bits of protein that carry the genetic code of an individual. Each child receives half of his or her genes from the mother and half from the father. Since both parents contribute to a child’s heredity, children tend to resemble both of their parents. Yet the number of genes is so great that it is virtually impossible for a brother and a sister to receive the same combination of genes from the mother and the father. Therefore, two brothers, two sisters, or a brother and a sister may resemble each other, but they cannot be identical to each other unless they are twins. Only twins can be identical, and only a certain type of twins. Identical twins result when a single fertilized ovum separates into two identical embryos during the first 14 days after fertilization. During pregnancy, the embryos develop into two identical babies. The babies are genetically identical because they inherited the same genes from the union of their mother’s single egg and their father’s single sperm. Identical twins look like Xerox copies of each other. Often, especially in the early months, they are indistinguishable, even to their parents. As the babies grow older, slight differences that help distinguish one twin from the other might become apparent. For instance, one twin might have a distinctive feature, such as a freckle, that the other twin lacks. As they mature, identical twins may continue to look so much alike that their friends and teachers confuse them. And no wonder, for identical twins not only look alike, they also tend to talk alike, walk alike, and even think alike. In their early years, identical twins share a virtually identical environment. They often occupy the same bed, eat the same food at the same time, play with the same toys, go to the same places, and interact with the same people. In addition, identical twins are united by an intimate relationship with each other. They are constant companions and enjoy a unique closeness. Some twins even seem to know what the other is thinking. It is not surprising, then, that identical twins resemble each other in behavior as well as appearance. But how much of this resemblance represents heredity and how much represents learning?

(Adapted from Lexis: Academic Vocabulary Study)

page | 114

Reading Comprehension

1. The term heredity in line 8 could be replaced by a) b) c) d)

tradition. cultural heritage. genetics. legacy.

2. The chances that siblings might receive exactly the same combination of genes from their parents is a) b) c) d)

highly likely. not unlikely. quite possible. hardly possible.

3. According to the text parents of identical twins usually can a) b) c) d)

identify each twin at birth easily and with certainty. identify the individual features of each child in the mother’s womb. detect differences between the children in the early months after birth. learn to distinguish one child from the other as the twins grow older.

4. The pronoun them in line 25 refers to a) b) c) d)

identical twins. the friends and teachers of identical twins. teachers of identical twins. parents, friends and teachers of identical twins.

5. Identical twins resemble each other in behavior and appearance because a) each twin always knows what the other is thinking. b) identical twins must sleep in the same bed and play with the same toys in their early years. c) each twin is not the other’s acquaintance. d) identical twins live in practically the same environment in their early years and tend to be very close.

page | 115

Mastering Proficiency Skills in English

1. CORRECT ANSWER:

c) genetics

The first sentences of the second paragraph provide clues to the meaning of heredity as it is used in line 8: Children inherit characteristics from their parents through genes, microscopic bits of protein that carry the genetic code of an individual. Each child receives half of his or her genes from the mother and half from the father. Since both parents contribute to a child’s heredity, children tend to resemble both of their parents.

2. CORRECT ANSWER:

d) hardly possible

The clue is in the fourth sentence of the second paragraph: Yet the number of genes is so great that it is virtually impossible for a brother and a sister to receive the same combination of genes from the mother and the father. The meaning of virtually impossible can be paraphrased as ‘impossible for all practical purposes’ or hardly possible, i.e. ‘certainly not possible’. The remaining three solutions offered all indicate a higher degree of likelihood, that is, possibility.

3. CORRECT ANSWER: d) learn to distinguish one child from the other as the twins grow older The third paragraph contains a number of clues: During pregnancy, the embryos develop into two identical babies … Often, especially in the early months, they are indistinguishable, even to their parents. As the babies grow older, slight differences that help distinguish one twin from the other might become apparent.

4. CORRECT ANSWER:

a) identical twins

The verb confuse in the sentence (lines 24-25) As they mature, identical twins may continue to look so much alike that their friends and teachers confuse them is a transitive verb (it is followed by the pronoun them, its direct object) and is used in the sense ‘to fail to differentiate from an often similar or related other’. The sentence can be paraphrased as As they mature, identical twins may continue to look so much alike that their friends and teachers fail to differentiate one twin from the other.

5. CORRECT ANSWER: d) identical twins live in practically the same environment in their early years and tend to be very close The clues to the answer which best completes the statement in question 5 is in the final paragraph: In their early years, identical twins share a virtually identical environment. In addition, identical twins are united by an intimate relationship with each other. They are constant companions and enjoy a unique closeness.

page | 116

Reading Comprehension

TEST 2 Hollywood’s true heroes have become redundant. Stuntmen and stuntwomen who entertained cinemagoers by falling from the sky, swimming with sharks and rolling fast cars have been replaced by technology. After surviving generations of screen fights, high falls and setting fire to themselves, the people behind the top actors’ most exciting scenes have had nearly all their work substituted by computer-generated stunts. Advances in special effects mean that the most dangerous and costly stunts can be achieved by mixing computer graphics with live action. This enables actors themselves to appear as if they are performing the stunts. The end of the stuntmen was signaled by blockbusters such as Volcano, starring Tommy Lee Jones, Titanic with Kate Winslet and The Lost World, Steven Spielberg’s sequel to Jurassic Park. Each is packed with computerized stunts. Wayne Michaels, one of Britain’s top stunt co-ordinators and the man who did a 250-meter bungee jump in the James Bond movie Goldeneye, said that at first studios tended to use computers for more extravagant stunts. “But they went on to do far more common and mundane things with computers, such as falling down the stairs,” he said. “As a result, stunt people are becoming extinct. It is not a popular view but it is a fact of life.” In the mid-1990s there were 12,000 registered stunt people, but more than half of them had difficulty finding work. Loren Janes, a co-founder of the Stuntmen’s Association of Motion Pictures and Television, said that, by 1997, teams of stuntmen and stuntwomen had found their work curtailed by technology. “Six or seven teams would be working on a film. Then, after a few days, the producers would come in and say, ‘You can go home. We’ve worked out how to do it on computer.’” The reason was simple: cost. Computer technology pioneered in movies such as Terminator 2 fell in price and became capable of creating stunts which would be either too expensive or too dangerous to attempt. One example was in Mission Impossible, starring Tom Cruise. In a scene where Cruise flies for 35 meters through the air from an exploding helicopter onto the back of a speeding train in the Channel tunnel, the image of the actor was simply superimposed on the scene using computers. A fall from 50 meters into water or an air bag can now be achieved for a tenth of the cost. With the rise of digital technology, insurance companies became more reluctant to cover genuine stunts. “If they know it can be done safely with visual effects, the companies will not insure real stunts,” said Simon Crane, a veteran stunt coordinator. Many in the industry believe stunt people should develop expertise in the new technology, acting as advisers on the virtual stunts. Otherwise, they will be left with little else to do than perform in the occasional fist fight. Some, however, think that stuntmen can survive in their traditional careers.

(Adapted from Cambridge First Certificate Reading)

page | 117

Mastering Proficiency Skills in English

1. The reflexive pronoun themselves in line 4 refers to a) b) c) d)

surviving generations. people following the top actors. stuntmen and stuntwomen. screen fights and high falls.

2. According to the text, advances in special effects make it possible for actors a) b) c) d)

to take part in stunts themselves. to seem to have performed the stunts themselves. to appear in stunts themselves. to perform stunts themselves.

3. The term common in line 13 could be replaced by a) b) c) d)

shared. rough. vulgar. ordinary.

4. Why is there less work for stuntmen and stuntwomen nowadays? a) More than six thousand stunt people could not find work easily in the mid-1990s. b) Studios use computer technology for more extravagant stunts only. c) The cost of stunts compared to the cost of technology for creating computerized stunts is higher. d) Insurance companies are always willing to cover genuine stunts. 5.

In the future stuntmen and stuntwomen a) b) c) d)

page | 118

shouldn’t insure themselves. should only perform fist fights. won’t become experts for virtual stunts. will have to master new skills.

Reading Comprehension

1. CORRECT ANSWER:

c) stuntmen and stuntwomen

The pronoun themselves appears in the sentence (lines 3-5) After surviving generations of screen fights, high falls and setting fire to themselves, the people behind the top actors’ most exciting scenes have had nearly all their work substituted by computer-generated stunts where it refers to the noun phrase the people behind the top actors’ most exciting scenes. The people referred to are stuntmen and stuntwomen.

2. CORRECT ANSWER:

b) to seem to have performed the stunts themselves

The clue to the answer which best completes the statement is in the final sentence of the first paragraph: This enables actors themselves to appear as if they are performing the stunts. In other words, actors do not actually perform, take part in or appear in stunts themselves thanks to advances in special effects. Instead, special effects create the illusion that actors had actually performed the stunts themselves.

3. CORRECT ANSWER:

d) ordinary

The adjective mundane is often used in the sense ‘commonplace’; common and mundane things could, then, be paraphrased as ordinary and commonplace things.

4. CORRECT ANSWER: c) The cost of stunts compared to the cost of technology for creating computerized stunts is higher Clues to the statement which best answers question 4 are given in the third paragraph: … by 1997, teams of stuntmen and stuntwomen had found their work curtailed by technology … The reason was simple: cost … A fall from 50 meters into water or an air bag can now be achieved for a tenth of the cost.

5. CORRECT ANSWER:

d) will have to master new skills

The answer is in the first sentence of the final paragraph: Many in the industry believe stunt people should develop expertise in the new technology, acting as advisers on the virtual stunts. That is, they will have to master new skills and broaden their knowledge in order to earn a living in their profession.

page | 119

Mastering Proficiency Skills in English

TEST 3 Most people tend to think of Americans as practical down-to-earth no-nonsense types, but I believe that what Americans like most is a good wide-ranging philosophical argument, plus a crisis. They have invented both for their national parks system. The crisis is that the parks are being, in the popular phrase, ‘loved to death’. Too many people are visiting the system – up to almost 400 million last year. It’s said they are ruining the plants with the pollution from their cars, scaring the animals, destroying by their numbers the wilderness experience the parks are supposed to offer. It’s hard for a visitor from Europe to feel this way. Three years ago we went to the Grand Tetons in Wyoming, which is my favorite park of all. Yes, you could find yourself waiting behind a line of cars as someone tried to photograph a herd of animals with a pocket camera. But then we drove a little way north, turned off the main road and found a small lake surrounded by fields of flowers, with the beautiful snow-capped range of the Tetons in the background. We saw a total of two other people during the whole long, sunny, perfect afternoon. The busiest park in the system, the Great Smoky Mountains between Tennessee and North Carolina, can get 60,000 visitors on a single summer’s day. That sounds plenty and it is, but all of these people are sharing an area only slightly smaller than the whole of Luxembourg, which has a permanent population seven times as great. Of course there are problems. For example, Yosemite has luxury houses commanding the finest views built for the executives of the company which owns all the park’s cafes and restaurants. There’s a video rental store now and even a small prison, for visitors who get drunk and disorderly. In spite of this, the main environmental threat to the best-known park in California is smog drifting east of Los Angeles. And so to the philosophical question. This takes many forms, but the basic argument is over how much should be done in the parks to satisfy human visitors. Should the accommodation be so basic that only true lovers of nature will be tempted to come? Or should it contain – as it increasingly does – en suite bathrooms and color TVs? Choose the former and you are necessarily excluding America’s growing population of old people, many of them desperate to enjoy a first experience of their own country’s beauty. And exactly what should be preserved? Twenty years ago Yellow Stone, perhaps the most famous park of all, decided to change to a ‘hands-off’ policy. Animals in danger of starving in the winter would be left to starve, just as nature intended. My advice is to stop trying too hard. Provide plenty of car parks and lodgings for visitors of all kinds. Ban radios and snowmobiles. But realize that for every thousand acres which are spoiled, there are a million which remain as beautiful as they were in George Washington’s day. (Taken from Cambridge First Certificate Reading)

page | 120

Reading Comprehension

1. The possessive pronoun their in line 6 refers to a) b) c) d)

cars. plants. animals. visitors.

2. Which is the most famous park in California? a) b) c) d)

The Grand Tetons. The Great Smoky Mountains. Yosemite. Yellow Stone.

3. Which adjective best describes what, according to the author, most people consider Americans to be like? a) b) c) d)

Superior. Sensible. Pretentious. Timid.

4. In line 26 former refers to a) b) c) d)

the philosophical question. how satisfied by national parks visitors should be. whether accommodation facilities should be attractive to nature lovers only. parks with luxurious facilities.

5. The author believes that a) a proportionally very large part of the American national parks system will remain unspoiled despite the development of tourism. b) tourists from Europe cannot enjoy the wilderness experience that American national parks are supposed to offer. c) people who visit American national parks often get drunk and disorderly. d) only true lovers of nature should be allowed to visit American national parks.

page | 121

Mastering Proficiency Skills in English

1. CORRECT ANSWER:

d) visitors

In order to understand what their in line 6 refers to, it is necessary to first understand what the pronoun they at the beginning of the same sentence refers to as the two pronouns have the same referent: It’s said they are ruining the plants with the pollution from their cars, scaring the animals, destroying by their numbers the wilderness experience the parks are supposed to offer. The clue is in the sentence immediately before this one: Too many people are visiting the system – up to almost 400 million last year. Both they in line 5 and their in line 6 refer to people visiting the American national parks system. People who visit the American national parks system can be referred to as visitors.

2. CORRECT ANSWER:

c) Yosemite

The clue is in the fourth paragraph: …Yosemite has luxury houses commanding the finest views… the main environmental threat to the best-known park in California is smog drifting east of Los Angeles. It is clear from the text that the phrase the best-known park in California describes Yosemite.

3. CORRECT ANSWER:

b) Sensible

The clue to the correct answer to question 3 is in the very first sentence of the text: Most people tend to think of Americans as practical down-to-earth no-nonsense types… The meaning of down-to-earth, no-nonsense, used to describe what most people consider Americans to be like, is ‘practical, tolerating no nonsense’. The meaning of sensible is very close to the meaning of practical.

4. CORRECT ANSWER: c) whether accommodation facilities should be attractive to nature lovers only A sense in which former is often used is ‘first in order of two or more things cited or understood’. The text makes it clear that the two things referred to are the two accommodation options cited immediately before the sentence in which former appears (lines 24-26): Should the accommodation be so basic that only true lovers of nature will be tempted to come? Or should it contain – as it increasingly does – en suite bathrooms and color TVs? The first of these refers to accommodation facilities which would be attractive to nature lovers only.

5. CORRECT ANSWER: a) a proportionally very large part of the American national parks system will remain unspoiled despite the development of tourism The clue to the answer which best completes the statement in question 5 can be found in the final sentence of the text: But realize that for every thousand acres which are spoiled, there are a million which remain as beautiful as they were in George Washington’s day. Careful reading of the final sentence of the first paragraph and the first sentence of the second paragraph, the fourth paragraph and the fifth paragraph reveals that the author does not hold the opinions expressed in (b), (c) and (d), respectively. page | 122

Reading Comprehension

TEST 4 For the first time, dictionary publishers are incorporating real spoken English into their data. It gives lexicographers (people who write dictionaries) access to a more vibrant, upto-date vernacular language which has never really been studied before. In one project, 150 volunteers each agreed to discreetly tie a Walkman recorder to their waist and leave it running for anything up to two weeks. Every conversation they had was recorded. When the data was collected, the length of tapes was 35 times the depth of the Atlantic Ocean. Teams of audio typists transcribed the tapes to produce the Spoken Corpus, a computerized database of ten million words. This has been the basis – along with an existing written corpus – for the Language Activator dictionary, described as “the book the world has been waiting for”. It shows advanced foreign learners of English how the language is really used. In the dictionary, key words such as “eat” are followed by related phrases, such as “wolf down” or “be a picky eater”, allowing the student to choose the appropriate phrase. “This kind of research would be impossible without computers,” said Delia Summers, a director of dictionaries. “It has transformed the way lexicographers work. If you look at the word ‘like’, you may intuitively think that the first and most frequent meaning is the verb, as in ‘I like swimming’. It is not. It is the preposition, as in: ‘she walked like a duck’. Just because a word or phrase is used doesn’t mean it ends up in a dictionary. The sifting out process is as vital as ever. But the database does allow lexicographers to search for a word and find out how frequently it is used – something that could only be guessed at intuitively before.” Researchers have found that written English works in a very different way to spoken English. The phrase ‘say what you like’ literally means ‘feel free to say anything you want’, but in reality it is used, evidence shows, by someone to prevent the other person voicing disagreement. The phrase ‘it’s a question of’ crops up in the database over and over again. It has nothing to do with enquiry, but it’s one of the most frequent English phrases which has never been in a language learner’s dictionary before: it is now. The Spoken Corpus computer shows how inventive and humorous people are when they are using language by twisting familiar phrases for effect. It also reveals the power of the pauses and noises we use to play for time, convey emotions, doubt and irony. For the moment, those benefiting most from the Spoken Corpus are foreign learners. “Computers allow lexicographers to search quickly through more examples of real English,” said Professor Geoffrey Leech of Lancaster University. “They allow dictionaries to be more accurate and give a feel for how language is being used.” The Spoken Corpus is part of the larger British National Corpus, an initiative carried out by several groups involved in the production of language learning materials: publishers, universities and the British Library. (Adapted from Cambridge Practice Tests for IELTS 1) page | 123

Mastering Proficiency Skills in English

1. The Spoken Corpus a) is being written up by Delia Summers, a director of dictionaries. b) is useful to those interested in how often a particular English word is used with a particular meaning. c) is a computerized database containing tens of millions of words. d) is larger than the British National Corpus. 2. The Language Activator dictionary is best described as being based on material from a) b) c) d)

taped conversations. taped conversations and the British Library. lexicographers’ taped conversations. written texts and taped conversations.

3. ‘Say what you like’ is frequently used in real-life conversations a) to indicate that the speaker really wants to know what the person he or she is talking to likes. b) when the speaker wants to make it clear that the person the speaker is talking to should not say he or she disagrees with the speaker. c) if the speaker wishes to allow the person to whom he or she is talking to speak in a loud voice in the course of a disagreement. d) to indicate that the person to whom the speaker is talking has the freedom to say whatever comes to mind. 4. The pronoun it in line 27 refers to a) b) c) d)

the database. English. the phrase. a language learner’s dictionary.

5. Which of the following could not be substituted for vital in as vital as ever in line 18? a) b) c) d)

page | 124

Important. Necessary. Active. Crucial.

Reading Comprehension

1. CORRECT ANSWER: b) is useful to those interested in how often a particular English word is used with a particular meaning Clues pointing to the correct answer to question 1 can be found throughout the text, in particular in paragraphs two and three. Careful reading of these two paragraphs shows that Deila Summers is not involved in writing up the Spoken Corpus. The final sentence of paragraph two shows that the database includes ten million, rather than tens of millions of words. Finally, the last paragraph makes it clear that the British National Corpus includes the Spoken Corpus and is, therefore, larger than it.

2. CORRECT ANSWER:

d) written texts and taped conversations

The final sentence of paragraph two and the first sentence of paragraph three provide clues to the correct answer to question 2: Teams of audio typists transcribed the tapes to produce the Spoken Corpus, a computerized database of ten million words. This has been the basis – along with an existing written corpus – for the Language Activator dictionary…

3. CORRECT ANSWER: b) when the speaker wants to make it clear that the person the speaker is talking to should not say he or she disagrees with the speaker. The clue is in the second sentence of the fourth paragraph: The phrase ‘say what you like’ literally means ‘feel free to say anything you want’, but in reality it is used, evidence shows, by someone to prevent the other person voicing disagreement. In this sentence, to prevent the other person voicing disagreement can be paraphrased as to stop the other person from saying they disagree.

4. CORRECT ANSWER:

c) the phrase

In the final sentence of the fourth paragraph, the pronoun it is used three times. In all three cases it refers to the phrase ‘it’s a question of’.

5. CORRECT ANSWER:

c) Active

The adjective vital is often used in the sense ‘of the utmost importance’, as in a vital clue or vital resources, in addition to the sense ‘full of life and vigor’. It is in the former sense that vital is used in line 18 of the text, meaning that as vital as ever can be paraphrased as as important as ever, as necessary as ever, as crucial as ever, but not as as active as ever.

page | 125

Mastering Proficiency Skills in English

TEST 5 Relatively recent archaeological finds have brought about a considerable change in perception about the Neanderthals. Neanderthals had previously been characterized more as primitive grunting beasts than as intelligent and compassionate human ancestors. However, evidence suggests that they may have exhibited more learned skills and social compassion than had previously been thought. The Neanderthals lived during a period that extended from at least 40,000 to 100,000 years ago. The Neanderthals differed from modern man in that they had a stronger and heavier skeleton and facial structure with a more projecting brow, a broader nose, and larger teeth. Casts made of Neanderthal brains by archaeologists show little difference in size from those of modern man. It has been known for some time that Neanderthals were rather skilled stone artisans. They are best known for their production of stone tools, which included a large number of scrapers and pointed implements. The techniques that Neanderthals used to prepare these tools demonstrated a clear and important technological advance over their predecessors. Edges of their stone tools have been studied under microscopes for evidence of how the tools may have been used. Many of the tools seem to have been used for working with wood, both for hacking at large branches and for doing more detailed work on smaller pieces; other tools were clearly used to prepare food, both meat and vegetables; still others, which resemble many of today’s suede and leather tools, were used to work with animal skins. A clearer picture of Neanderthals has come about recently as archaeologists have determined that, in addition to the known ability to develop and employ tools in a rather skilled way, Neanderthals also exhibited evidence of beliefs and social rituals, aspects of life that were newly introduced by Neanderthals and that provide evidence of humanlike thoughts and feelings. Neanderthal cemeteries have been discovered in places like La Ferrassie in France and Shanidar in Iraq; Neanderthal remains in these cemeteries have provided proof of social organization and ritual in the Neanderthals. One skeleton of a Neanderthal was found with a crushed skull; the blow on the top of the head, perhaps from a falling boulder, had quite obviously been the cause of death. What was interesting was that study of the skeleton showed that while he had been alive this man had been seriously handicapped with a defect that had limited use of the upper right side of his body, that he suffered from arthritis, and that he was blind in one eye. The fact that he had survived well into old age was a strong indication that others had been helping to care for him and to provide him with food rather than allowing him to die because he was no longer fit. Other skeletal remains of Neanderthals show clear examples of burial rituals. These discoveries about Neanderthals help to create a picture of Neanderthals as beings with the feelings and emotions that go along with developed social customs and rituals.

(Taken from Longman Preparation Course for the TOEFL test) page | 126

Reading Comprehension

1. Which of the following would be the best evidence that Neanderthals possessed humanlike feelings? a) b) c) d)

The discovery of a Neanderthal skeleton with a crushed skull. The discovery that some Neanderthals were handicapped. The discovery that Neanderthals could survive well into old age. The discovery that Neanderthals seem to have cared for those unable to care for themselves.

2. The possessive pronoun their in line 14 refers to a) b) c) d)

techniques. Neanderthals. tools. edges.

3. Which of the following does the author not mention as an activity for which Neanderthals used tools? a) Woodworking. b) Making clothing out of animal skins. c) Food preparation. d) Cutting branches. 4. Which of the following could be substituted for remains in line 25? a) b) c) d)

Skeletons. Stays. Leftovers. Ruins.

5. The author argues that Neanderthals a) b) c) d)

were as primitive as had been thought in the past. had significantly smaller brains than modern man. had no beliefs. are known to have produced pointed tools.

page | 127

Mastering Proficiency Skills in English

1. CORRECT ANSWER: d) The discovery that Neanderthals seem to have cared for those unable to care for themselves Although the four discoveries in (a) through (d) are all mentioned in the text, only the one given in (d) is mentioned, in the fourth paragraph, as evidence that Neanderthals possessed humanlike feelings.

2. CORRECT ANSWER:

b) Neanderthals

The phrase their predecessors in line 14 could be paraphrased as their ancestors. Therefore, the pronoun their must refer to the Neanderthals, rather than to techniques, tools or edges.

3. CORRECT ANSWER:

b) Making clothing out of animal skins

Careful reading of the final sentence of the third paragraph makes it clear that although the author mentions the use of tools to work with animal skins, he does not specify for what purpose or purposes these animal skins were used: …still others, which resemble many of today’s suede and leather tools, were used to work with animal skins.

4. CORRECT ANSWER:

a) Skeletons

A clue to the meaning of remains in line 25, ‘parts of the body that are left after the person has died’, is given in the immediately following sentence in the text: One skeleton of a Neanderthal was found with a crushed skull… where one skeleton of a Neanderthal refers to one of the Neanderthal remains found in Neanderthal cemeteries.

5. CORRECT ANSWER:

d) are known to have produced pointed tools

The clue to the correct answer to question 5 is in the second sentence of the third paragraph: They are best known for their production of stone tools, which included a large number of scrapers and pointed implements. One of the senses in which the noun implement is used is ‘tool’. Careful reading of the final sentence of the first paragraph, the final sentence of the second paragraph and the first sentence of the fourth paragraph shows that the author does not advance the arguments in (a), (b) and (c), respectively.

page | 128

WRITING Composition writing has become a crucial and inevitable segment in language assessment. It displays both the student’s competence in written expression and his/her mastery of English language. In addition, it enables the students to fully grasp the meaning of evaluation, raising their awareness in self-correction strategies by exposing them to the relevant criteria.

Mastering Proficiency Skills in English

The grading criteria marks a PASSING or FAILING grade based on the following three components and their presence/absence in the compositions: 1. Content-related ideas are organized in the fashion of a simple composition, having a beginning, middle and concluding section. 2. Ideas are coherently presented, without digressions, illustrating a logical flow. 3. Fair grammatical competence is presented and the text is not difficult to understand. The following composition samples have been evaluated according to the abovementioned criteria. During the exam at the Faculty of Philology and Arts the candidates were able to choose ONE out of the three topics provided: 1. My favorite place to relax 2. I will always remember this movie because it tells an excellent story 3. Mobile phones have become very important To familiarize the candidates with the evaluation methods four sample compositions have been provided, two of which received a passing grade and two of which failed to demonstrate the skills required for the B2 level writing. Besides the underlined sections that illustrate grammatical, as well as content flaws, a brief evaluation comment has been presented following each composition.

page | 130

Writing

1. My favorite place to relax My favourite place to relax is my grandmother’s house in a small place called Virovo. Virovo is so beautiful a place and I’m loving it! People are relaxed and normal – you’ll definitely agree that’s very rare gift among people nowadays. Granny’s house is little and has got modest furniture, but it has got something very important – that spot has got ‘soul’. That’s the place which every single person could call ‘a real house’. It has got everything the one house should have – kitchen, living-room, bathroom and a lovely garden where I’m usually spending my free time. I always like to go there – no matter how I fell, whether I’m sad or happy… or angry… or in love with someone… It seems to me like there all problems simply dissapear and happyness grows with each spent hour. The best is when summer comes and school is over – then I go there with my best friend and we’re making parties every night. Over the winter is very nice, too – Christmas tree, presents… granny always prepares great meals for New Year and Christmas – she’s really great cook! Sometimes when I’m bored (which happens rarely when I’m there), I simply grab the blanket, lie in the corner of the room and watch the old videos. I’m cheerful and satisfied whenever I go there and that’s why the granny’s house in Virovo is definitely my favourite place to relax. EVALUATION: The essay satisfies the required length. 1. It is closely related to the topic, treating relevant ideas and perceptions. It clearly illustrates the beginning, middle and concluding sections content-wise. 2. Coherence is evident and the necessary transition words and expressions are applied. It provides a logical idea flow without slipping off the topic. 3. Minor grammar mistakes (evident misuse of the Present Continuous Tense; omission of articles, as well as occasional inappropriate use of articles and spelling) do not present a serious obstacle in understanding of the text. EVALUATED: Pass

page | 131

Mastering Proficiency Skills in English

2. I will always remember this movie because it tells an excellent story My favourite movie is “I’m not a girl, not yet a woman” with B. Spears. That movie played and exellent story, love story. In that film she had advanture which every girl want to have in life. It’s a story about life and love, about friends and about your dreams. She is going to jorney, on that jorney she had met a her love. Later, she had a problem because him with her father. She must to make a choise to father and love. After everything, she is coming to him and be happy, the happiest girl in the world. The story from this film showed us what is the most important in life. Like everybody says “Love is the stronges!” I’ll remember this movie because telling me this messages which is correct.

EVALUATION: This essay is brief and not detailed enough to illustrate the topic. 1. The writing is related to the topic but not organized well and with unclear sections. In addition, the required analysis of ‘the excellent story’ is not accomplished. 2. Although sentences are related to the topic there is an absence of the logical flow of ideas and the sentences are not connected. The candidate simply wrote ideas down randomly, not considering the development of ideas beforehand. 3. Grammatical mistakes (punctuation, tenses, verbal constructions, articles and prepositions) additionally create a problem in understanding the text. EVALUATED: Fail

page | 132

Writing

3. Mobile phones have become very important They have become very important in our society. Every member of that society has one. Even the children. Of course, it is good to have them around, especially in case of a need. If we need some help or we must call someone for any reason at all, we don’t have to go anywhere. We just take out our mobile phone right out of our pockets and call a friend. Also, it is good for parents to know where their children are. But, unfortunately, it has a bad side as well. So much radiation, which is caused by these devices is bad for our health. Also, you can hear annoying ringing from it everywhere, everytime. But, it seems that they are one of most important devices of today’s society. EVALUATION: Although it seems brief, this essay provides a full development of the topic. 1. Completely content-related and clearly organized into a brief discussion on the issue. It is composed of a general introduction, a body and a conclusion. 2. The idea flow is evident and the body is clearly divided into positive and negative aspects of the issue, followed by explanations and examples. 3. Grammatical mistakes are almost completely absent and minor (mostly problems with reference). This candidate writes skillfully and clearly and therefore it is easy to understand the main points of the text. EVALUATED: Pass

page | 133

Mastering Proficiency Skills in English

4. My favorite place to relax I have never had some specific place to relax. I love to relax with my boyfriend in his house in the woods, in park with my friends, at my grandmothers house, but watching films with my boyfriend I love the most. That’s the best relaxation for me. We drink Coce, smoke cigaretts, eat candies, we enjoy ourselves totally. And at the end of the evening we have pilow fights. Then we go for a walk… We are eating icecream allmost every evening. Every day by his side is a relaxation for me. EVALUATION: The composition is too brief and underdeveloped in terms of content. 1. The essay is topic relevant, however absolutely without prior consideration of the organization of ideas. The whole structure is not in a form of a composition. 2. The absence of the logical idea flow and evident jumping from one topic to another illustrate a conversational style. No transitional devices are used, whatsoever. It is difficult to detect the primary idea and the aim of the essay. The focus of the idea shifts constantly. 3. Although there are no major grammatical mistakes (mostly spelling, punctuation, a possessive form, one tense form and inappropriate article use), the direct translation from Serbian shows that the candidate is unfamiliar with Englishlike expressions and constructions. The idea development is extremely unsatisfactory and fails to transmit the point of the writing. EVALUATED: Fail

page | 134

Writing

SUGGESTED ESSAY TOPICS The entrance exam offers three different topics, allowing the candidates to choose the most suitable one. These topics include elements of a descriptive, narrative and argumentative composition. We present here a list of such various topics that enable the candidates to prepare for the experience of composition writing in different lexical fields, practicing a variety of sentence constructions and tense forms. Essay topics: 1. 2. 3. 4. 5. 6. 7. 8. 9. 10. 11. 12. 13. 14. 15. 16. 17. 18. 19. 20.

This was my family nest as a child If I could turn back time… Describe the place(s) you don’t enjoy visiting and explain your choice Love makes the world go round He/she is a strange person How important is Facebook in our lives? I wish I lived in… The importance of music in our lives A summary of a recently read book A favorite relative/family member After-school activities I enjoy If I won the lottery… The healthy/unhealthy food I consume Never in my life have I felt such relief! My neighborhood is a funny place Topics I like to discuss with my peers My city/town could look much better if… Common problems with parents My favorite piece of jewelry/clothing The means of transportation I would never use

page | 135

Further Practice

FURTHER PRACTICE Grammar exercises 1. Thomson, A. J. & Martinet, A. V. (1986). A Practical English Grammar Exercises 1 & 2. Oxford: Oxford University Press. 2. Murphy, R. (1994). English Grammar in Use. Cambridge: Cambridge University Press. 3. Paunović, T., Marković, Lj. & Savić, M. (2004). English Department Entrance Exam: Tips and Tricks. Niš: Filozofski fakultet. 4. Popović, Lj. & Popović, M. (1996). Gramatika engleskog jezika kroz testove. Beograd: Zavet. 5. Rosset, E. (1992). 3000 tests. Irun: Editorial Stanley.

Vocabulary items and confusable words 1. Thomas, B. & Matthews, L. (2007). Vocabulary for First Certificate. Cambridge: Cambridge University Press. 2. Redman, S. (2003). English Vocabulary in Use (pre-intermediate and intermediate). Cambridge: Cambridge University Press. 3. Clark, S. & Pointon, G. (2003). Word for word. Oxford: Oxford University Press.

Reading comprehension 1. Thomas, B. & Matthews, L. (2007). Vocabulary for First Certificate. Cambridge: Cambridge University Press. 2. Roberts, P. (1994). Cambridge First Certificate Reading. Cambridge: Cambridge University Press. 3. O’Neill, R., Duckworth, M. & Gude, K. (1997). New Success at First Certificate. Oxford: Oxford University Press.

Composition 1. O’Neill, R., Duckworth, M. & Gude, K. (1997). New Success at First Certificate. Oxford: Oxford University Press. 2. Hopper, V. F., Gale, C. & Griffith, B. W. (2000). Essentials of Writing. New York: Barron’s Educational Series, Inc. page | 136

References

REFERENCES 1. Alexander, L. G. (1990). Longman English Grammar Practice. Harlow: Longman Group UK Limited. 2. Burgmeier, A., Eldred, G. & Zimmerman, C. B. (1991). LEXIS: academic vocabulary study. New Jersey: Prentice-Hall, Inc. 3. Collins Cobuild English Dictionary. (1993). London: Harper Collins Publishers. 4. Čubrović, B. & Daničić, M. (2005). English Entrance Exam Practice 3. Beograd: Philologia. 5. Eastwood, J. (2005). Grammar Finder. Oxford: Oxford University Press. 6. Hewings, M. (1999). Advanced Grammar in Use. Cambridge: Cambridge University Press. 7. Hlebec, B. (2003). Gramatika engleskog jezika za srednje škole. Beograd: Zavod za udžbenike i nastavna sredstva. 8. Hogue, A. (2003). The Essentials of English: A Writer’s Handbook. New York: Pearson Education, Inc. 9. Hopper, V. F., Gale, C. & Griffith, B. W. (2000). Essentials of Writing. New York: Barron’s Educational Series, Inc. 10. Jakeman, V. & McDowell, C. (1996). Cambridge Practice Tests for IELTS 1. Cambridge: Cambridge University Press. 11. Longman Dictionary of Contemporary English. (2001). Harlow: Pearson Education Limited. 12. Longman Language Activator. (1993). Harlow: Longman Group UK Limited. 13. MacMillan English Dictionary for Advanced Students. (2002). Oxford: Macmillan Publishers Limited. 14. O’Malley, J. M. & Pierce, L. V. (1996). Authentic Assessment for English Language Learners. Reading, MA: Addison-Wesley Publishing Company. 15. Oxford Collocations Dictionary 4th edition. (2003). Oxford: Oxford University Press. 16. Phillips, D. (2005). Longman Preparation Course for the TOEFL test: next generation iBT. New York: Pearson / Longman. 17. Popović, Lj. & Mirić, V. (1999). Gramatika engleskog jezika sa vežbanjima. Beograd: Zavet. 18. Roberts, P. (1994). Cambridge First Certificate Reading. Cambridge: Cambridge University Press. 19. Swan, M. (2005). Practical English Usage. Oxford: Oxford University Press. 20. Thomson, A. J. & Martinet, A. V. (1992). A Practical English Grammar. Oxford: Oxford University Press. page | 137